Re: [R] Replace columns in a data.frame randomly splitted

2011-12-01 Thread agent dunham


The thing is that I've already been working with df1, and I was looking for
a function that could replace values knowing rows and columns. Does it
exist?

Thank you, u...@host.com

--
View this message in context: 
http://r.789695.n4.nabble.com/Replace-columns-in-a-data-frame-randomly-splitted-tp4122926p4127405.html
Sent from the R help mailing list archive at Nabble.com.

__
R-help@r-project.org mailing list
https://stat.ethz.ch/mailman/listinfo/r-help
PLEASE do read the posting guide http://www.R-project.org/posting-guide.html
and provide commented, minimal, self-contained, reproducible code.


Re: [R] Error adding Bigmemory package

2011-12-01 Thread Uwe Ligges

On 30.11.2011 21:53, RogerP wrote:

I am trying to add the bigmemory packages but I get the following error
message:

In file included from bigmemory.cpp:14:0:
../inst/include/bigmemory/isna.hpp: In function 'bool neginf(double)':
../inst/include/bigmemory/isna.hpp:22:57: error: 'isinf' was not declared in
this scope
make: *** [bigmemory.o] Error 1
ERROR: compilation failed for package 'bigmemory'
* removing '/usr/local/lib/R/library/bigmemory'
* restoring previous '/usr/local/lib/R/library/bigmemory'

The downloaded packages are in
 '/tmp/RtmpNrkt2a/downloaded_packages'
Updating HTML index of packages in '.Library'
Warning message:
In install.packages(bigmemory) :
   installation of package 'bigmemory' had non-zero exit status

install.packages(gsl)

Error in install.packages(gsl) : object 'gsl' not found

install.packages(gnulib)

Error in install.packages(gnulib) : object 'gnulib' not found


I am using the GNU compiler version 4.5.1 from blastwave on a Solaris 10
box.
I can't seem to find how to compile bigmembory successfully.  I can complile
and add
other packages.



See

http://cran.r-project.org/web/checks/check_results_bigmemory.html

and find it also fails the Solaris based CRAN checks.

Hence please report to the package maintainer directly, since reports 
from CRAN maintainers and the CRAN results are obviously ignored.


Best,
Uwe Ligges






roger

--
View this message in context: 
http://r.789695.n4.nabble.com/Error-adding-Bigmemory-package-tp4124863p4124863.html
Sent from the R help mailing list archive at Nabble.com.

__
R-help@r-project.org mailing list
https://stat.ethz.ch/mailman/listinfo/r-help
PLEASE do read the posting guide http://www.R-project.org/posting-guide.html
and provide commented, minimal, self-contained, reproducible code.


__
R-help@r-project.org mailing list
https://stat.ethz.ch/mailman/listinfo/r-help
PLEASE do read the posting guide http://www.R-project.org/posting-guide.html
and provide commented, minimal, self-contained, reproducible code.


[R] References for book R In Action by Kabacoff

2011-12-01 Thread Ravi Kulkarni
I know this is not really an R question - it is a query about a recent book
on R (R In Action) by Robert Kabacoff, (Manning Publications 2011).
There are many references to interesting topics in R in the book, BUT, I do
not find a bibliography/list of references in the book!  
Does anybody know if there are errata for the book available some place?

Thanks,
  Ravi

--
View this message in context: 
http://r.789695.n4.nabble.com/References-for-book-R-In-Action-by-Kabacoff-tp4127625p4127625.html
Sent from the R help mailing list archive at Nabble.com.

__
R-help@r-project.org mailing list
https://stat.ethz.ch/mailman/listinfo/r-help
PLEASE do read the posting guide http://www.R-project.org/posting-guide.html
and provide commented, minimal, self-contained, reproducible code.


Re: [R] References for book R In Action by Kabacoff

2011-12-01 Thread Uwe Ligges

On 01.12.2011 10:10, Ravi Kulkarni wrote:

I know this is not really an R question - it is a query about a recent book
on R (R In Action) by Robert Kabacoff, (Manning Publications 2011).
There are many references to interesting topics in R in the book, BUT, I do
not find a bibliography/list of references in the book!
Does anybody know if there are errata for the book available some place?


I'd ask the author!

Uwe Ligges




Thanks,
   Ravi

--
View this message in context: 
http://r.789695.n4.nabble.com/References-for-book-R-In-Action-by-Kabacoff-tp4127625p4127625.html
Sent from the R help mailing list archive at Nabble.com.

__
R-help@r-project.org mailing list
https://stat.ethz.ch/mailman/listinfo/r-help
PLEASE do read the posting guide http://www.R-project.org/posting-guide.html
and provide commented, minimal, self-contained, reproducible code.


__
R-help@r-project.org mailing list
https://stat.ethz.ch/mailman/listinfo/r-help
PLEASE do read the posting guide http://www.R-project.org/posting-guide.html
and provide commented, minimal, self-contained, reproducible code.


[R] Odp: R code

2011-12-01 Thread Petr PIKAL
Hi
 
 Hi everybody,
 
 I am unable to resolve this error using the following for loop. Would
 appreciate help. The same loop works with for(i in 1:92) strangely. I
 checked the .raw input file and everything is kosher, even Line 547
 mentioned in the error message.
 
 I wonder if there is any problem with the paste function.

I do not believe it. In that case the message will be, that R can not open 
the file.

You speak about input file but actually you are reading 93 files. Which 
one is wrong? Did you check that one? And how did you checked it?

There could be some unseen characters in this row. Without some more info 
I doubt you get any better advice.

Petr

 
 Thanks very much in advance.
 
 
 **
 for(i in 1:93)
 {
 inputdta- read.table(file=gsub( ,,paste(JSR_network_[,i,].RAW),
 fixed=TRUE), header=TRUE)
 colnames(inputdta) - c(jsrid,firmid,jsrstart,injsr)
 print(nrow(inputdta))
 }
 
 ***
 Message: Error in scan(file, what, nmax, sep, dec, quote, skip, nlines,
 na.strings, : line 547 did not have 4 elements
 
 
 --
 View this message in context: http://r.789695.n4.nabble.com/R-code-
 tp4125904p4125904.html
 Sent from the R help mailing list archive at Nabble.com.
[[alternative HTML version deleted]]
 
 __
 R-help@r-project.org mailing list
 https://stat.ethz.ch/mailman/listinfo/r-help
 PLEASE do read the posting guide 
http://www.R-project.org/posting-guide.html
 and provide commented, minimal, self-contained, reproducible code.

__
R-help@r-project.org mailing list
https://stat.ethz.ch/mailman/listinfo/r-help
PLEASE do read the posting guide http://www.R-project.org/posting-guide.html
and provide commented, minimal, self-contained, reproducible code.


Re: [R] how big (in RAM and/or disk storage) is each of these objects in a list?

2011-12-01 Thread Uwe Ligges


On 26.11.2011 20:36, John wrote:

On Sat, 26 Nov 2011 12:41:08 -0600
Paul Johnsonpauljoh...@gmail.com  wrote:


Greetings, friends (and others :) )

We generated a bunch of results and saved them in an RData file. We
can open, use, all is well, except that the size of the saved file is
quite a bit larger than we expected.  I suspect there's something
floating about in there that one of the packages we are using puts in,
such as a spare copy of a data frame that is saved in some subtle way
that has escaped my attention.

Consider a list of objects. Are there ways to do these things:

1. ask R how much memory is used by the things inside the list?



A bit late, but hopefully still helpful:


See ?object.size

and particularly note

 Associated space (e.g. the environment of a function and what the
 pointer in a ‘EXTPTRSXP’ points to) is not included in the
 calculation.



2.   Does as.expression(anObject) print everything in there? Or, is
there a better way to convert each thing to text or some other format
that you can actually read line by line to see what is in there, to
see everything?


See ?dump

and also carefully read the notes.




If there's no giant hidden data frame floating about, I figure I'll
have to convert symmetric matrices to lower triangles or such to save
space.  Unless R already is automatically saving a matrix in that way
but just showing me the full matrix, which I suppose is possible. If
you have other ideas about general ways to make saved objects smaller,
I'm open for suggestions.


Look carefully for environments attached to one or more of the objects.

Best,
Uwe Ligges







As an initial step, what is the result of running ls() with your RData
file loaded?  You should get a list of what is in memory.  Using RData
files can be as space-efficient or costly as the user's habits.  Did you
use save() or the save.image() command to produce the file? The
save.image() command stashes what is in memory and if you've run a
number of experimental procedures that did not pan out and you did not
discard with the results with rm(), they were saved to the rdata file
along with the information you did want, a procedure rather like filing
away all your work in a file drawer and then emptying the waste basket
into the drawer as well. If you save the data with ascii = TRUE as an
option, you can troll through the file and read what you saved.

JWD

__
R-help@r-project.org mailing list
https://stat.ethz.ch/mailman/listinfo/r-help
PLEASE do read the posting guide http://www.R-project.org/posting-guide.html
and provide commented, minimal, self-contained, reproducible code.


__
R-help@r-project.org mailing list
https://stat.ethz.ch/mailman/listinfo/r-help
PLEASE do read the posting guide http://www.R-project.org/posting-guide.html
and provide commented, minimal, self-contained, reproducible code.


Re: [R] R code

2011-12-01 Thread Jim Holtman
the problem might be a missing quote.  Try 

quote = ''

Or it might be a comment character, so try

comment.char = ''

This happens a lot if your data is not clean

Sent from my iPad

On Nov 30, 2011, at 19:48, nsaraf nsa...@gmail.com wrote:

 )

__
R-help@r-project.org mailing list
https://stat.ethz.ch/mailman/listinfo/r-help
PLEASE do read the posting guide http://www.R-project.org/posting-guide.html
and provide commented, minimal, self-contained, reproducible code.


Re: [R] Drawing ticks in the 3rd and 4th row of a lattice

2011-12-01 Thread Ashim Kapoor
Dear Mr Sarkar,

My apologies for not being articulate enough. Your solution worked
perfectly.

Many thanks,
Ashim

On Tue, Nov 22, 2011 at 9:12 PM, Deepayan Sarkar
deepayan.sar...@gmail.comwrote:

 On Fri, Nov 18, 2011 at 11:22 AM, Ashim Kapoor ashimkap...@gmail.com
 wrote:
  Dear all,
 
  I want to draw ticks on the 3rd and 4th row of a lattice. How do I do
 this
  ? In my search of the help, I discovered a parameter alternating,which
 kind
  of says where the ticks will be but does not suffice for me.

 You need to explain more clearly what you want. Your plot does already
 have ticks in the 3rd and 4th row. If you need the x-axes labeled in
 each row, you need to have something like

 scales=list(x = list(relation = free, rot = 45, ...

 -Deepayan

  I am running this command : -
 
  barchart(X03/1000~time|Company,
  data=df1[which(df1$time!=1),],
  horiz=F,
 
 
 scales=list(x=list(rot=45,labels=paste(Mar,c(07,08,09,10,11
  ,par.strip.text=list(lineheight=1,lines=2,cex=.75),
  ylab = In Rs.
  Million,xlab=,layout=c(3,4),as.table=T,between=list(y=1))
 
 
  where my data is  : -
 
  dput(df1)
  structure(list(Company = structure(c(9L, 7L, 1L, 6L, 8L, 4L,
  2L, 5L, 11L, 10L, 9L, 7L, 1L, 6L, 8L, 4L, 2L, 5L, 11L, 10L, 9L,
  7L, 1L, 6L, 8L, 4L, 2L, 5L, 11L, 10L, 9L, 7L, 1L, 6L, 8L, 4L,
  2L, 5L, 11L, 10L, 9L, 7L, 1L, 6L, 8L, 4L, 2L, 5L, 11L, 10L, 9L,
  7L, 1L, 6L, 8L, 4L, 2L, 5L, 11L, 10L), .Label = c(Bharat Petroleum
 Corpn.
  Ltd.,
  Chennai Petroleum Corpn. Ltd., Company Name, Essar Oil Ltd.,
  Hindalco Industries Ltd., Hindustan Petroleum Corpn. Ltd.,
  Indian Oil Corpn. Ltd., Mangalore Refinery  Petrochemicals Ltd.,
  Reliance Industries Ltd., Steel Authority Of India Ltd.,
  Sterlite Industries (India) Ltd.), class = factor), time = c(7,
  7, 7, 7, 7, 7, 7, 7, 7, 7, 8, 8, 8, 8, 8, 8, 8, 8, 8, 8, 9, 9,
  9, 9, 9, 9, 9, 9, 9, 9, 10, 10, 10, 10, 10, 10, 10, 10, 10, 10,
  11, 11, 11, 11, 11, 11, 11, 11, 11, 11, 1, 1, 1, 1, 1, 1, 1,
  1, 1, 1), X03 = c(722931.1, 751620.5, 304456.3, 294868.9, 192712.6,
  36695.4, 188313.4, 98954.9, 100088.7, 72379.9, 848517.5, 864562.2,
  347310.9, 301022.1, 253514.5, 165661.6, 206377.7, 108897, 109336.3,
  71207.6, 1003504.6, 1145993.8, 392261.5, 341086, 289737.4, 359837.2,
  252964.3, 90036.2, 90474.8, 127623.2, 1411082.1, 907480.4, 364637.5,
  290915.7, 255397.4, 328557.2, 202855.3, 118725.4, 116647.6, 106254.9,
  1772254.7, 1204856.9, 469935.6, 313527.6, 320131.1, 384323.5,
  260813.9, 137403.3, 137238.5, 136888.4, 1151658, 974902.76, 375720.36,
  308284.06, 262298.6, 255014.98, 64.92, 110803.36, 110757.18,
  102870.8)), row.names = c(Reliance Industries Ltd..7, Indian Oil
 Corpn.
  Ltd..7,
  Bharat Petroleum Corpn. Ltd..7, Hindustan Petroleum Corpn. Ltd..7,
  Mangalore Refinery  Petrochemicals Ltd..7, Essar Oil Ltd..7,
  Chennai Petroleum Corpn. Ltd..7, Hindalco Industries Ltd..7,
  Sterlite Industries (India) Ltd..7, Steel Authority Of India Ltd..7,
  Reliance Industries Ltd..8, Indian Oil Corpn. Ltd..8, Bharat
 Petroleum
  Corpn. Ltd..8,
  Hindustan Petroleum Corpn. Ltd..8, Mangalore Refinery  Petrochemicals
  Ltd..8,
  Essar Oil Ltd..8, Chennai Petroleum Corpn. Ltd..8, Hindalco
 Industries
  Ltd..8,
  Sterlite Industries (India) Ltd..8, Steel Authority Of India Ltd..8,
  Reliance Industries Ltd..9, Indian Oil Corpn. Ltd..9, Bharat
 Petroleum
  Corpn. Ltd..9,
  Hindustan Petroleum Corpn. Ltd..9, Mangalore Refinery  Petrochemicals
  Ltd..9,
  Essar Oil Ltd..9, Chennai Petroleum Corpn. Ltd..9, Hindalco
 Industries
  Ltd..9,
  Sterlite Industries (India) Ltd..9, Steel Authority Of India Ltd..9,
  Reliance Industries Ltd..10, Indian Oil Corpn. Ltd..10, Bharat
  Petroleum Corpn. Ltd..10,
  Hindustan Petroleum Corpn. Ltd..10, Mangalore Refinery 
 Petrochemicals
  Ltd..10,
  Essar Oil Ltd..10, Chennai Petroleum Corpn. Ltd..10, Hindalco
  Industries Ltd..10,
  Sterlite Industries (India) Ltd..10, Steel Authority Of India
 Ltd..10,
  Reliance Industries Ltd..11, Indian Oil Corpn. Ltd..11, Bharat
  Petroleum Corpn. Ltd..11,
  Hindustan Petroleum Corpn. Ltd..11, Mangalore Refinery 
 Petrochemicals
  Ltd..11,
  Essar Oil Ltd..11, Chennai Petroleum Corpn. Ltd..11, Hindalco
  Industries Ltd..11,
  Sterlite Industries (India) Ltd..11, Steel Authority Of India
 Ltd..11,
  Reliance Industries Ltd..1, Indian Oil Corpn. Ltd..1, Bharat
 Petroleum
  Corpn. Ltd..1,
  Hindustan Petroleum Corpn. Ltd..1, Mangalore Refinery  Petrochemicals
  Ltd..1,
  Essar Oil Ltd..1, Chennai Petroleum Corpn. Ltd..1, Hindalco
 Industries
  Ltd..1,
  Sterlite Industries (India) Ltd..1, Steel Authority Of India Ltd..1
  ), .Names = c(Company, time, X03), reshapeLong = structure(list(
 varying = structure(list(X03 = c(X03.07, X03.08, X03.09,
 X03.10, X03.11, X03.1)), .Names = X03, v.names = X03, times
 =
  c(7,
 8, 9, 10, 11, 1)), v.names = X03, idvar = Company, timevar =
  time), .Names = c(varying,
  v.names, idvar, timevar)), class = data.frame)
 
  

Re: [R] Non-finite finite-difference value error in eha's aftreg

2011-12-01 Thread Milan Bouchet-Valat
Le mercredi 30 novembre 2011 à 18:41 +0100, Göran Broström a écrit :
 On Wed, Nov 16, 2011 at 3:06 PM, Milan Bouchet-Valat
 nalimi...@club.fr wrote:
 Hi list!
 
 
 I'm getting an error message when trying to fit an accelerated
 failure
 time parametric model using the aftreg() function from package
 eha:
  Error in optim(beta, Fmin, method = BFGS, control =
 list(trace =
as.integer(printlevel)),  :
  non-finite finite-difference value [2]
 
 This only happens when adding four specific covariates at the
 same time
 in the model (see below). I understand that kind of problem
 can come
 from a too high correlations between my covariates, but is
 there
 anything I can do to avoid it? 
 
 Yes, remove one (or more) covariate(s). The design matrix is almost
 certainly singular. 
Ah, thanks, that's what I feared. ;-)

What do you mean, though, by almost? Is there a way to measure that?

 Does something need to be improved in aftreg.fit?
 
 Yes, error messages. I'll look at it.  
Cool. It's always good to get a clear message saying that you're asking
for something impossible, so that you don't try to fix it (like I did).


Regards


 My data set is constituted of 34,505 observations (years) of
2,717
 individuals, which seems reasonable to me to fit a complex
model like
 that (covariates are all factors with less than 10 levels). 
 I can send
 it by private mail if somebody wants to help debugging this.
 
 The details of the model and errors follow, but feel free to
ask for
 more testing. I'm using R 2.13.1 (x86_64-redhat-linux-gnu),
eha 2.0-5
 and survival 2.36-9.
 
 
 Thanks for your help!
 
 
  m -aftreg(Surv(start, end, event) ~ homo1 + sexego +
dipref1
 ++ t.since.school.q,
 +data=ms, dist=loglogistic, id=ident)
 Error in optim(beta, Fmin, method = BFGS, control =
list(trace =
 as.integer(printlevel)),  :
  non-finite finite-difference value [2]
 Calls: aftreg - aftreg.fit - aftp0 - optim
 
  traceback()
 4: optim(beta, Fmin, method = BFGS, control = list(trace =
 as.integer(printlevel)),
   hessian = TRUE)
 3: aftp0(printlevel, ns, nn, id, strata, Y, X, offset, dis,
means)
 2: aftreg.fit(X, Y, dist, strats, offset, init, shape, id,
control,
   center)
 1: aftreg(Surv(start, end, event) ~ homo1 + sexego + dipref1 +
   t.since.school.q, data = ms, dist = loglogistic, id =
ident)
 
 __
 R-help@r-project.org mailing list
 https://stat.ethz.ch/mailman/listinfo/r-help
 PLEASE do read the posting guide
http://www.R-project.org/posting-guide.html
 and provide commented, minimal, self-contained, reproducible
code.
 
 
 
 -- 
 Göran Broström


__
R-help@r-project.org mailing list
https://stat.ethz.ch/mailman/listinfo/r-help
PLEASE do read the posting guide http://www.R-project.org/posting-guide.html
and provide commented, minimal, self-contained, reproducible code.


Re: [R] x, y for point of intersection

2011-12-01 Thread Hans W Borchers
Monica has sent me some data and code for taking a quick look. As it
turned out, there was a simple programming error on her side. The
segm_distance() function in package 'pracma' is working correctly. And
there is no minimization procedure in here, it simply solves some
equations from plane geometry.

Maybe the function suggested by Don is faster, I haven't checked that.

Regards,  Hans Werner


2011/11/29 Monica Pisica pisican...@hotmail.com

 Hi again,

 Working with my real data and not with the little example i sent to the list 
 i discovered that segm_distance function from package pracma does not 
 converge to 0 in all cases, even if i increase the number of iteration to 
 10,000 for example. It seems that it depends on the initialization point - 
 most like a minimization function.

 So my thanks  go to Don who's suggestion works for the real data as well 
 without any problems - so far ;-) He suggested to use the function 
 crossing.psp from package spatstat.

 Thanks again to all who have answered and helped to solve my problem. I 
 certainly learned few new things.

 Monica





  From: macque...@llnl.gov
  To: pisican...@hotmail.com
  CC: r-help@r-project.org
  Date: Wed, 23 Nov 2011 14:03:42 -0800

  Subject: Re: [R] x, y for point of intersection
 
  The function crossing.psp() in the spatstat package might be of use.
  Here's an excerpt from its help page:
 
  crossing.psp package:spatstat R Documentation
  Crossing Points of Two Line Segment PatternsDescription:
  Finds any crossing points between two line segment patterns.
  Usage:
  crossing.psp(A,B)
  -Don
 
 
  --
  Don MacQueen
 
  Lawrence Livermore National Laboratory
  7000 East Ave., L-627
  Livermore, CA 94550
  925-423-1062
 


__
R-help@r-project.org mailing list
https://stat.ethz.ch/mailman/listinfo/r-help
PLEASE do read the posting guide http://www.R-project.org/posting-guide.html
and provide commented, minimal, self-contained, reproducible code.


Re: [R] SPSS - R

2011-12-01 Thread Yvonnick Noel

Dear Kristi,

Also. can anyone recommend any resources to help SPSS users learn to things in 
R?


You may want to have a look at the R2STATS package, a simple GUI for 
linear models.


Best,

Yvonnick Noel
University of Brittany
Department of Psychology
Rennes, France

__
R-help@r-project.org mailing list
https://stat.ethz.ch/mailman/listinfo/r-help
PLEASE do read the posting guide http://www.R-project.org/posting-guide.html
and provide commented, minimal, self-contained, reproducible code.


[R] permutate elements in a vector

2011-12-01 Thread Wendy
Hi all, 

I have a vector, e.g., A = c(10, 20, 30, 40). This 4 numbers have 4! = 24
different combination. I want to generate a matrix with one combination in
one row, so the output would be 

B = 
10 20 30 40 
10 40 20 30
... 

Does anyone know how to do this easily in R? Thank you very much.

Wendy 

--
View this message in context: 
http://r.789695.n4.nabble.com/permutate-elements-in-a-vector-tp4127821p4127821.html
Sent from the R help mailing list archive at Nabble.com.

__
R-help@r-project.org mailing list
https://stat.ethz.ch/mailman/listinfo/r-help
PLEASE do read the posting guide http://www.R-project.org/posting-guide.html
and provide commented, minimal, self-contained, reproducible code.


[R] Raster manipulation in R

2011-12-01 Thread jjcabrera20
Hello everyone in the forum
For introducing myself I would say I have a basic knowledge of R.
Now I am intended to implement a flood algorithm using R. I have some MatLab
experience doing these, and as an example to explain more or less what I
want, I have a m code to calculate the slope from a Digital elevation model:

slope=zeros(row,col);
for i=2:row-1
for j=2:col-1
   dzdx=(raster(i,j+1)-raster(i,j-1))/(2*res);
   dzdy=(raster(i+1,j)-raster(i-1,j))/(2*res);
   slope(i,j)=sqrt(dzdx^2+dzdy^2);
end
end;

The question is to know how to do the similar procedure on R. All
suggestions are welcome
Thanks

All best,

Jorge
PD:I am using R on windows system 64 bits


--
View this message in context: 
http://r.789695.n4.nabble.com/Raster-manipulation-in-R-tp4127768p4127768.html
Sent from the R help mailing list archive at Nabble.com.

__
R-help@r-project.org mailing list
https://stat.ethz.ch/mailman/listinfo/r-help
PLEASE do read the posting guide http://www.R-project.org/posting-guide.html
and provide commented, minimal, self-contained, reproducible code.


[R] combining arima and ar function

2011-12-01 Thread Samir Benzerfa
Hi everyone

 

I've got a problem regarding the arima() and the ar() function for
autoregressive series. I would simply like to combine them. 

 

To better understand my question, I first show you how I'm using these two
functions individually (see file in the attachement).

 

1)  apply(TSX,2, function(x) ar(na.omit(x),method=mle)$order
# this function finds the optimal (autoregressive) order for each series
(briefly speaking: for each series I get a specific number)

2)  apply(TSX,2,function(x)arima(x,order=c(1,0,0))$residuals
# this function puts an autoregressive model of order 1 on each series

 

 

What I'd like to do, is to combine them, such that the resulting orders
(numbers) from the first function are used as orders in the second function.
So in the specific example attached the results from the first function are
6,5,1 and these numbers should be used as the order in function two. I tried
to simply put the two functions together as follows:

 

apply(TSX,2,function(x) arima(x,order=c((apply(TSX,2,
function(x)ar(na.omit(x),method=mle)$order))),0,0))

 

However, I get the error message  'order' must be a non-negative numeric
vector of length 3.

 

Any hints? I hope you can help me with this issue.

 

Many thanks and best regards, S.B.

 

 

 

__
R-help@r-project.org mailing list
https://stat.ethz.ch/mailman/listinfo/r-help
PLEASE do read the posting guide http://www.R-project.org/posting-guide.html
and provide commented, minimal, self-contained, reproducible code.


[R] Resampling with replacement on a binary (0, 1) dataset to get Cis

2011-12-01 Thread lincoln
...is it possible to do that?

I apologize for something that must be a very trivial question for most of
you but, unfortunately, it is not for me.
A binary variable is measured, say, 50 times each year during 10 year. My
interest is focused on the percentage of 1s with respect to the total if
each year.
There is no way to repeat those measure within each year and getting the CIs
by the normal way.
By the way, it would be important to get even a rough estimate of the CIs of
these estimates (/n/1//n/1+/n/0).

In case this is not a blasphemy, how might be done in R?

Thanks in advance for any help

--
View this message in context: 
http://r.789695.n4.nabble.com/Resampling-with-replacement-on-a-binary-0-1-dataset-to-get-Cis-tp4127990p4127990.html
Sent from the R help mailing list archive at Nabble.com.

__
R-help@r-project.org mailing list
https://stat.ethz.ch/mailman/listinfo/r-help
PLEASE do read the posting guide http://www.R-project.org/posting-guide.html
and provide commented, minimal, self-contained, reproducible code.


Re: [R] R endnote entry

2011-12-01 Thread hwright
1. May I suggest that you try Mendeley for bibliographic management
2. There are also citations for specific R packages when you download each
version.

Let's use the ade4 package as an example:
http://cran.r-project.org/web/packages/ade4/index.html
if you selection citation info the corresponding bibtex input has been
provided.

 @Article{,
title = {The ade4 package: implementing the duality diagram for
  ecologists},
author = {S. Dray and A.B. Dufour},
journal = {Journal of Statistical Software},
year = {2007},
volume = {22},
pages = {1-20},
number = {4},
  }

This is a reliable way to deal with references in other programs ie:
LaTeX...etc. I save all my references as bibtex into mendeley and then have
a subfolders for R analysis/methods that contain the package citations.

Using a .txt (bibtex file made from the ade4 package citation), my inserted
citation in a word document from Mendeley looks like this: (R Development
Core Team, 2011)
and the formal reference in the bibliography is: R Development Core Team.
2011. R: A Language and Environment for Statistical Computing.

If however, you are still determined to use endnote, then you have 2 ways to
configure your reference,  manually add a reference by copy/paste the
correct title from the citation, etc. Then configure your .ens style for the
appropriate journal, etc. If you insert the citation and it becomes R.D.C.T
then it's likely the original input is fine and you have to configure your
style output.

good luck!




-
---
Heather A. Wright, PhD candidate
Ecology and Evolution of Plankton
Stazione Zoologica Anton Dohrn
Villa Comunale
80121 - Napoli, Italy
--
View this message in context: 
http://r.789695.n4.nabble.com/R-endnote-entry-tp4126826p4127866.html
Sent from the R help mailing list archive at Nabble.com.

__
R-help@r-project.org mailing list
https://stat.ethz.ch/mailman/listinfo/r-help
PLEASE do read the posting guide http://www.R-project.org/posting-guide.html
and provide commented, minimal, self-contained, reproducible code.


[R] efficient ways to dynamically grow a dataframe

2011-12-01 Thread Matteo Richiardi
Hi,
I'm trying to write a small microsimulation in R: that is, I have a
dataframe with info on N individuals for the base-year and I have to
grow it dynamically for T periods:

df = data.frame(
 id = 1:N,
 x =
)

The most straightforward way to solve the problem that came to my mind
is to create for every period a new dataframe:

for(t in 1:T){
 for(i in 1:N){
  row = data.frame(
   id = i,
   t = t,
   x = ...
   )
   df = rbind(df,row)
 }
}

This is very inefficient and my pc gets immediately stucked as N is
raised above some thousands.
As an alternative, I created an empty dataframe for all the projected
periods, and then filled it:

df1 = data.frame(
 id = rep(1:N,T),
 t = rep(1:T, each = N),
 x = rep(NA,N*T)
)

for(t in 1:T){
 for(i in 1:N){
  x = ...
  df1[df1$id==i  df1$t==t,x] = x
 }
}
df = rbind(df,df1)

This is also too slow, and my PC gets stucked. I don't want to go for
a matrix, because I'd loose the column names and everything will
become too much error-prone.
Any suggestions on how to do it?
Thanks in advance,
Matteo




-- 
Matteo Richiardi
University of Turin
Faculty of Law
Department of Economics Cognetti De Martiis
via Po 53, 10124 Torino
Email: matteo.richia...@unito.it
Tel. +39 011 670 3870
Web page: http://www.personalweb.unito.it/matteo.richiardi/

__
R-help@r-project.org mailing list
https://stat.ethz.ch/mailman/listinfo/r-help
PLEASE do read the posting guide http://www.R-project.org/posting-guide.html
and provide commented, minimal, self-contained, reproducible code.


Re: [R] I cannot get species scores to plot with site scores in MDS when I use a distance matrix as input. Problems with NA's?

2011-12-01 Thread Edwin
Dear Gavin,
Thanks a lot for your reply. I am still not sure if your suggestion can help
me or not because I have got different types of errors while trying it. I
have tried my best to troubleshoot them and made some progress, but I have
reached a dead end (I don’t know how to go on). Below I am copping my TinnR
script. It contains not only the R script but also the errors returned in
the R console. I apologize that the script is still not really reproducible
but I just do not know how to create a sample data frame for it. Thanks
again for your help.  Edwin.
##Load required packages##
library(vegan)
library(MASS)
library (FD)
library(dummies)
##Load full table of species traits and taxonomic data##
AllTraits-read.delim(SpeciesTraitsFullTbl.txt,header=T)
##Edit data##
row.names(AllTraits)-AllTraits$species.code
TraitsNMDS-AllTraits[,c(8:12,14:18,23,25,30)]
##Check variable class using function allClass from
http://gettinggeneticsdone.blogspot.com/2010   
/08/quickly-find-class-of-dataframe-vectors.html##
allClass - function(x) {unlist(lapply(unclass(x),class))} 
allClass(TraitsNMDS)
##Change variables to fulfill requirements for gowdis() function##
  #Define ordinal variables#
  TraitsNMDS$leaf.type-ordered(TraitsNMDS$leaf.type, levels =
c(simple, pinnate, bipinnate))
  TraitsNMDS$dispersal.rank-ordered(TraitsNMDS$dispersal.rank, levels =
c(s_disp,m_disp,l_disp))
#coerce integer an binary variables to numeric# 
TraitsNMDS$max.height-as.numeric(TraitsNMDS$max.height)
TraitsNMDS$heterospory-as.numeric(TraitsNMDS$heterospory)
##Test wrapper function##
wrapper- function(x, method, ...) {gowdis(x, ...)}
WrapTest1-metaMDS(TraitsNMDS, distfun = wrapper, x=TraitsNMDS, asym.bin =
heterospory, ord = podani)
##Returned error: 'Error in if (any(autotransform, noshare  0, wascores) 
any(comm  0)) { :   missing value where TRUE/FALSE needed'##
  #Set autotransform, noshare and wascores to FALSE (even if TRUE is
desired) to see if problems is with metaMDS function#
  WrapTest2-metaMDS(TraitsNMDS, distfun = wrapper, x=TraitsNMDS, asym.bin =
heterospory, ord= podani, autotransform =FALSE, noshare = FALSE,
wascores = FALSE)
  #Returned error: 'Error in FUN(X[[1L]], ...) : only defined on a data
frame with all numeric variables'#
  #transform factors to dummies and numeric#
  TraitsNMDSCompleteDumm-dummy.data.frame(TraitsNMDSComplete,
c(leaf.type,dispersal.rank))
 
TraitsNMDSCompleteDumm$leaf.typesimple-as.numeric(TraitsNMDSCompleteDumm$leaf.typesimple)
 
TraitsNMDSCompleteDumm$leaf.typepinnate-as.numeric(TraitsNMDSCompleteDumm$leaf.typepinnate)
 
TraitsNMDSCompleteDumm$leaf.typebipinnate-as.numeric(TraitsNMDSCompleteDumm$leaf.typebipinnate)
 
TraitsNMDSCompleteDumm$dispersal.ranks_disp-as.numeric(TraitsNMDSCompleteDumm$dispersal.ranks_disp)
 
TraitsNMDSCompleteDumm$dispersal.rankm_disp-as.numeric(TraitsNMDSCompleteDumm$dispersal.rankm_disp)
 
TraitsNMDSCompleteDumm$dispersal.rankl_disp-as.numeric(TraitsNMDSCompleteDumm$dispersal.rankl_disp)
  
##Re-test wrapping function##
WrapTest3-metaMDS(TraitsNMDSCompleteDumm, distfun = wrapper,
x=TraitsNMDSCompleteDumm,asym.bin =heterospory, ord = podani)
  #The function runs but returns an error related to the gowdis() function
after the transformation of the data is done:  
  #'Square root transformation Wisconsin double standardization. Error in
gowdis(x, ...) : w needs to be a numeric vector of length = number of
variables in x'
 # But testing the wrapper alone does work!#
DistMatrixWrapper-wrapper(x=TraitsNMDSCompleteDumm,asym.bin =
heterospory, ord = podani)
class(DistMatrixWrapper)
[1] dist
 # I do not know why the error is produced


--
View this message in context: 
http://r.789695.n4.nabble.com/I-cannot-get-species-scores-to-plot-with-site-scores-in-MDS-when-I-use-a-distance-matrix-as-input-Pr-tp4103699p4127949.html
Sent from the R help mailing list archive at Nabble.com.

__
R-help@r-project.org mailing list
https://stat.ethz.ch/mailman/listinfo/r-help
PLEASE do read the posting guide http://www.R-project.org/posting-guide.html
and provide commented, minimal, self-contained, reproducible code.


Re: [R] Bayesian Quantile regression installation

2011-12-01 Thread Sarah Goslee
Please send your questions to the R-help list, and not directly to me.

You provide your OS this time, but not any of the other information
requested in the posting guide, like your version of R.

What you did would be nice too. Used the menu option? Or used
install.packages() at the command line?

If this is an internet-enabled computer, the latter is by far
the easiest option: it will download and install the package
for you.

On Thu, Dec 1, 2011 at 6:34 AM, kalam narendar reddy
narendarcse...@gmail.com wrote:
 Dear Sarah Goslee,
 The following i sthe error if i tried to use the install packages from
 local zip files ven though i have loacated the bayesQR_1.3 which is
  winRAR zip archieve  fie.
  Error in read.dcf(file.path(pkgname, DESCRIPTION), c(Package, Type))
 :
   cannot open the connection
 In addition: Warning message:
 In read.dcf(file.path(pkgname, DESCRIPTION), c(Package, Type)) :
   cannot open compressed file 'bayesQR_1.3(1)/DESCRIPTION', probable reason
 'No such file or directory'

The filename shown here is not the one that's in the CRAN archive -
did you try to download it more than once? That may confuse Windows, I
don't know. I don't use Windows (another reason you shouldn't be
emailing me directly). But try renaming it without the (1) and see if
that works.

 tell me what is the reason for it. i have tried to install the package from
 local zip file(binary version for windows of bayesian quantile regression
 ).my os is windows7 and i have followed the procedure as it is told in the
 manual for windows for package installation.still i am unanle to install.pls
 understand my problem.
 thanks in advance for your kind reply.pls reply as early as possible.


Sarah

__
R-help@r-project.org mailing list
https://stat.ethz.ch/mailman/listinfo/r-help
PLEASE do read the posting guide http://www.R-project.org/posting-guide.html
and provide commented, minimal, self-contained, reproducible code.


[R] nipals in the chemometrics package in R

2011-12-01 Thread zz dd
Hello
i need some precision about nipals in the chemometrics package in R .

When i use nipals in chemometrics i obtain T and P matrix.

I really don't understand what to do with these two matrix to obtain
the scores for every the component (like in spss fo example)

 Comp1Comp2   Comp3
quest1 0,8434  0,54333   0,3466
quest2 0,665   0,7655  0,433

Thank you very much for your help
(I know that X=TP+E)... But don't understand else

__
R-help@r-project.org mailing list
https://stat.ethz.ch/mailman/listinfo/r-help
PLEASE do read the posting guide http://www.R-project.org/posting-guide.html
and provide commented, minimal, self-contained, reproducible code.


[R] How to speed up int2bin conversion?

2011-12-01 Thread Jonas Jägermeyr
Dear R-help members,

I'm processing a large amount of MODIS data where quality assessment 
information is stored as an integer value for each pixel. I have to 
converted this number to an 8 digit binary flag to get access to the 
stored quality code (e.g. in2bin(165,8) = 1 0 1 0 0 1 0 1).

Unfortunately, I did not manage to find a package providing a fast 
function to do so. I need to run this on millions of pixels and thus 
wrote the following function.

int2bin - function(x,ndigits){
 base - array(NA, dim=c(length(x), ndigits))
 for(q in 1:ndigits){
   base[, ndigits-q+1] - (x %% 2)
   x - (x %/% 2)
   }
 bin- apply(base,1,paste,collapse=)
 return(bin)
}

Since it is still slow, I have to find a way to express this more 
elegantly. I'd really appreciate any help.

Thanking you, with best regards

Jonas Jägermeyr

Potsdam Institute for Climate Impact Research
Research Domain II

[[alternative HTML version deleted]]

__
R-help@r-project.org mailing list
https://stat.ethz.ch/mailman/listinfo/r-help
PLEASE do read the posting guide http://www.R-project.org/posting-guide.html
and provide commented, minimal, self-contained, reproducible code.


Re: [R] FIML with missing data in sem package

2011-12-01 Thread John Fox
Dear Dustin,

 -Original Message-
 From: r-help-boun...@r-project.org [mailto:r-help-bounces@r-
 project.org] On Behalf Of Dustin Fife
 Sent: November-30-11 9:51 PM
 To: r-help@r-project.org
 Subject: [R] FIML with missing data in sem package
 
 Is there a way to use full information maximum likelihood (FIML) to
 estimate missing data in the sem package? For example, suppose I have a
 dataset with complete information on X1-X3, but missing data (MAR) on
 X4.
 Is there a way to use FIML in this case? I know lavaan and openmx allow

No, the sem package doesn't handle missing data. You could, however, use
another package, such as mice, for multiple imputation of the missing data.

Best,
 John


John Fox
Senator William McMaster
  Professor of Social Statistics
Department of Sociology
McMaster University
Hamilton, Ontario, Canada
http://socserv.mcmaster.ca/jfox


 you to do it, but I couldn't find anything in the documentation for the
 sem package. Thanks!
 
 --
 Dustin Fife
 Graduate Student
 Quantitative Psychology
 University of Oklahoma
 
   [[alternative HTML version deleted]]
 
 __
 R-help@r-project.org mailing list
 https://stat.ethz.ch/mailman/listinfo/r-help
 PLEASE do read the posting guide http://www.R-project.org/posting-
 guide.html
 and provide commented, minimal, self-contained, reproducible code.

__
R-help@r-project.org mailing list
https://stat.ethz.ch/mailman/listinfo/r-help
PLEASE do read the posting guide http://www.R-project.org/posting-guide.html
and provide commented, minimal, self-contained, reproducible code.


[R] bigmemory on Solaris

2011-12-01 Thread Jay Emerson
At one point we might have gotten something working (older version?) on
Solaris x86, but were never successful on Solaris sparc that I remember --
it isn't a platform we can test and support.  We believe there are problems
with BOOST library compatibilities.

We'll try (again) to clear up the other warnings in the logs, though.  !-)

We should also revisit the possibility of a CRAN BOOST library for use by a
small group of packages (like bigmemory) which might make patches to BOOST
easier to track and maintain.  This might improve things in the long run.

Jay

-- 
John W. Emerson (Jay)
Associate Professor of Statistics
Department of Statistics
Yale University
http://www.stat.yale.edu/~jay

[[alternative HTML version deleted]]

__
R-help@r-project.org mailing list
https://stat.ethz.ch/mailman/listinfo/r-help
PLEASE do read the posting guide http://www.R-project.org/posting-guide.html
and provide commented, minimal, self-contained, reproducible code.


Re: [R] permutate elements in a vector

2011-12-01 Thread Berend Hasselman

Wendy wrote
 
 Hi all, 
 
 I have a vector, e.g., A = c(10, 20, 30, 40). This 4 numbers have 4! = 24
 different combination. I want to generate a matrix with one combination in
 one row, so the output would be 
 
 B = 
 10 20 30 40 
 10 40 20 30
 ... 
 
 Does anyone know how to do this easily in R? Thank you very much.
 
 Wendy
 

You want permutations not combinations.

library(e1071)
permutations(4) * 10

or 

library(gtools)
A - 10*(1:4)
permutations(4,4,A)

Berend


--
View this message in context: 
http://r.789695.n4.nabble.com/permutate-elements-in-a-vector-tp4127821p4128227.html
Sent from the R help mailing list archive at Nabble.com.

__
R-help@r-project.org mailing list
https://stat.ethz.ch/mailman/listinfo/r-help
PLEASE do read the posting guide http://www.R-project.org/posting-guide.html
and provide commented, minimal, self-contained, reproducible code.


[R] how to get inflection point in binomial glm

2011-12-01 Thread René Mayer

Dear All,

I have a binomial response with one continuous predictor (d) and one  
factor (g) (8 levels dummy-coded).


glm(resp~d*g, data, family=binomial)

Y=b0+b1*X1+b2*X2 ... b7*X7

how can I get the inflection point per group, e.g., P(d)=.5

I would be grateful for any help.

Thanks in advance,
René

__
R-help@r-project.org mailing list
https://stat.ethz.ch/mailman/listinfo/r-help
PLEASE do read the posting guide http://www.R-project.org/posting-guide.html
and provide commented, minimal, self-contained, reproducible code.


Re: [R] Raster manipulation in R

2011-12-01 Thread Jean V Adams
jjcabrera20 wrote on 12/01/2011 04:02:47 AM:

 Hello everyone in the forum
 For introducing myself I would say I have a basic knowledge of R.
 Now I am intended to implement a flood algorithm using R. I have some 
MatLab
 experience doing these, and as an example to explain more or less what I
 want, I have a m code to calculate the slope from a Digital elevation 
model:
 
 slope=zeros(row,col);
 for i=2:row-1
 for j=2:col-1
dzdx=(raster(i,j+1)-raster(i,j-1))/(2*res);
dzdy=(raster(i+1,j)-raster(i-1,j))/(2*res);
slope(i,j)=sqrt(dzdx^2+dzdy^2);
 end
 end;
 
 The question is to know how to do the similar procedure on R. All
 suggestions are welcome
 Thanks
 
 All best,
 
 Jorge
 PD:I am using R on windows system 64 bits


If I am interpreting your code correctly (I don't use MatLab myself), 
something like this should give you the same result in R:

# example matrix of heights
raster - matrix(runif(20, 10, 30), nrow=4, ncol=5)

# example resolution
res - 8.5

# dimensions of matrix
drast - dim(raster)

# for every non-boundary point in the matrix,
# calculate the distances between its adjacent columns (dzdx) and rows 
(dzdy)
dzdx - raster[2:(drast[1]-1), 3:drast[2]] - raster[2:(drast[1]-1), 
1:(drast[2]-2)]
dzdy - raster[3:drast[1], 2:(drast[2]-1)] - raster[1:(drast[1]-2), 
2:(drast[2]-1)]

# calculate the slope from these distances and the resolution
slope - sqrt(dzdx^2 + dzdy^2) / (2*res)


Jean
[[alternative HTML version deleted]]

__
R-help@r-project.org mailing list
https://stat.ethz.ch/mailman/listinfo/r-help
PLEASE do read the posting guide http://www.R-project.org/posting-guide.html
and provide commented, minimal, self-contained, reproducible code.


Re: [R] simplify source code

2011-12-01 Thread Christof Kluß

Hi

now I'd like to do

for (colname in c('ColName1','ColName2','ColName3')) {  
dat - measurements$colname

But that does not work, though I can write

measurements$C1 (same as measurements$C1)
(but different to measurements[C1]!)

Can you give me a hint?

greetings
Christof



Am 26-11-2011 23:30, schrieb Christof Kluß:

Hi

I would like to shorten

mod1 - nls(ColName2 ~ ColName1, data = table, ...)
mod2 - nls(ColName3 ~ ColName1, data = table, ...)
mod3 - nls(ColName4 ~ ColName1, data = table, ...)
...

is there something like

cols = c(ColName2,ColName3,ColName4,...)

for i in ...
mod[i-1] - nls(ColName[i] ~ ColName1, data = table, ...)

I am looking forward to help

Christof



__
R-help@r-project.org mailing list
https://stat.ethz.ch/mailman/listinfo/r-help
PLEASE do read the posting guide http://www.R-project.org/posting-guide.html
and provide commented, minimal, self-contained, reproducible code.


Re: [R] Resampling with replacement on a binary (0, 1) dataset to get Cis

2011-12-01 Thread David Winsemius


On Dec 1, 2011, at 6:34 AM, lincoln wrote:


...is it possible to do that?

I apologize for something that must be a very trivial question for  
most of

you but, unfortunately, it is not for me.
A binary variable is measured, say, 50 times each year during 10  
year. My
interest is focused on the percentage of 1s with respect to the  
total if

each year.
There is no way to repeat those measure within each year and getting  
the CIs

by the normal way.
By the way, it would be important to get even a rough estimate of  
the CIs of

these estimates (/n/1//n/1+/n/0).

In case this is not a blasphemy, how might be done in R?


?prop.test



Thanks in advance for any help



David Winsemius, MD
West Hartford, CT

__
R-help@r-project.org mailing list
https://stat.ethz.ch/mailman/listinfo/r-help
PLEASE do read the posting guide http://www.R-project.org/posting-guide.html
and provide commented, minimal, self-contained, reproducible code.


Re: [R] permutate elements in a vector

2011-12-01 Thread Jean V Adams
Wendy wrote on 12/01/2011 04:27:09 AM:

 Hi all, 
 
 I have a vector, e.g., A = c(10, 20, 30, 40). This 4 numbers have 4! = 
24
 different combination. I want to generate a matrix with one combination 
in
 one row, so the output would be 
 
 B = 
 10 20 30 40 
 10 40 20 30
 ... 
 
 Does anyone know how to do this easily in R? Thank you very much.
 
 Wendy 


This has been asked and answered in earlier posts.  Search R-Help for 
permutations.  Here is one solution (see below).

Jean


http://tolstoy.newcastle.edu.au/R/e8/help/09/10/0983.html

From: David Winsemius 
Date: Sat, 10 Oct 2009

require(combinat) 
permn(c(23,46,70,71,89))

[[alternative HTML version deleted]]

__
R-help@r-project.org mailing list
https://stat.ethz.ch/mailman/listinfo/r-help
PLEASE do read the posting guide http://www.R-project.org/posting-guide.html
and provide commented, minimal, self-contained, reproducible code.


Re: [R] How to speed up int2bin conversion?

2011-12-01 Thread jim holtman
If we assume that you are just convert 8 bits, here is one way with a
table lookup.  If more than 8 bits, just partition the data and
repeat.  This sets up a mapping table one time for the lookup.  Does
1M in 0.3 seconds on my computer; is this fast enough?

 # initialize a matrix with 8 bit binary values
 # one time
 require(bitops)
 b2c - character(256)
 for (i in 0:255){
+ b2c[i + 1] - sprintf(%1d%1d%1d%1d%1d%1d%1d%1d
+ , bitAnd(i, 0x80) != 0
+ , bitAnd(i, 0x40) != 0
+ , bitAnd(i, 0x20) != 0
+ , bitAnd(i, 0x10) != 0
+ , bitAnd(i, 0x8) != 0
+ , bitAnd(i, 0x4) != 0
+ , bitAnd(i, 0x2) != 0
+ , bitAnd(i, 0x1) != 0
+ )
+ }

 # create vector with 1M values
 x - as.integer(1:1e6)

 int2bin - function(val)
+ {
+ b2c[bitAnd(val, 0xff) + 1]
+ }

 system.time(int2bin(x))
   user  system elapsed
   0.310.000.32

On Thu, Dec 1, 2011 at 7:14 AM, Jonas Jägermeyr jonas...@pik-potsdam.de wrote:
 Dear R-help members,

 I'm processing a large amount of MODIS data where quality assessment
 information is stored as an integer value for each pixel. I have to
 converted this number to an 8 digit binary flag to get access to the
 stored quality code (e.g. in2bin(165,8) = 1 0 1 0 0 1 0 1).

 Unfortunately, I did not manage to find a package providing a fast
 function to do so. I need to run this on millions of pixels and thus
 wrote the following function.

 int2bin - function(x,ndigits){
     base - array(NA, dim=c(length(x), ndigits))
     for(q in 1:ndigits){
           base[, ndigits-q+1] - (x %% 2)
           x - (x %/% 2)
       }
     bin- apply(base,1,paste,collapse=)
     return(bin)
 }

 Since it is still slow, I have to find a way to express this more
 elegantly. I'd really appreciate any help.

 Thanking you, with best regards

 Jonas Jägermeyr

 Potsdam Institute for Climate Impact Research
 Research Domain II

        [[alternative HTML version deleted]]


 __
 R-help@r-project.org mailing list
 https://stat.ethz.ch/mailman/listinfo/r-help
 PLEASE do read the posting guide http://www.R-project.org/posting-guide.html
 and provide commented, minimal, self-contained, reproducible code.




-- 
Jim Holtman
Data Munger Guru

What is the problem that you are trying to solve?
Tell me what you want to do, not how you want to do it.

__
R-help@r-project.org mailing list
https://stat.ethz.ch/mailman/listinfo/r-help
PLEASE do read the posting guide http://www.R-project.org/posting-guide.html
and provide commented, minimal, self-contained, reproducible code.


Re: [R] Problem installing the package tkrplot

2011-12-01 Thread Marcos Amaris Gonzalez
Thanks Peter. I was knowing that R had not found the header tk.h file.
But i did not where it was. I opened my Manage Packages Synaptic and
installed tk-dev and other similar packages, and voila! It work. I
coul install tkrplot and the other packages that i was needing.

THANKS, for your answer, This is part of your Linux distribution.

On Wed, Nov 30, 2011 at 9:08 PM, peter dalgaard pda...@gmail.com wrote:

 On Nov 30, 2011, at 15:31 , Marcos Amaris Gonzalez wrote:

 Hello people. My problem is that when I try to install the package
 tkrplot, I got the next problem:

 You seem to be missing the tk.h file. This is, most likely, in a tcl/tk 
 development package that you haven't installed. This is part of your Linux 
 distribution, not R. (You're not telling us which Linux you are running, and 
 it can't be pinpointed any further without that information, but look for 
 something like tk-devel and tcl-devel.)


 install.packages(tkrplot)
 Installing package(s) into ‘/home/marcos/R/i686-pc-linux-gnu-library/2.13’
 (as ‘lib’ is unspecified)
 probando la URL
 'http://www.vps.fmvz.usp.br/CRAN/src/contrib/tkrplot_0.0-23.tar.gz'
 Content type 'application/x-gzip' length 39037 bytes (38 Kb)
 URL abierta
 ==
 downloaded 38 Kb

 * installing *source* package ‘tkrplot’ ...
 configure: creating ./config.status
 config.status: creating src/Makevars
 ** libs
 gcc -I/usr/share/R/include -I/usr/include/tcl8.5 -I/usr/include/tcl8.5
    -fpic  -std=gnu99 -O3 -pipe  -g -c tcltkimg.c -o tcltkimg.o
 tcltkimg.c:2:16: fatal error: tk.h: No existe el fichero o el directorio
 compilation terminated.
 make: *** [tcltkimg.o] Error 1
 ERROR: compilation failed for package ‘tkrplot’
 * removing ‘/home/marcos/R/i686-pc-linux-gnu-library/2.13/tkrplot’
 Warning in install.packages :
  installation of package 'tkrplot' had non-zero exit status

 The downloaded packages are in
       ‘/tmp/RtmpHRfZ1k/downloaded_packages’

 In this moment I have R version 2.13.2 (2011-09-30). I stayed
 thankfull for the any help.

 ---
 Marcos Amaris Gonzalez

 __
 R-help@r-project.org mailing list
 https://stat.ethz.ch/mailman/listinfo/r-help
 PLEASE do read the posting guide http://www.R-project.org/posting-guide.html
 and provide commented, minimal, self-contained, reproducible code.

 --
 Peter Dalgaard, Professor,
 Center for Statistics, Copenhagen Business School
 Solbjerg Plads 3, 2000 Frederiksberg, Denmark
 Phone: (+45)38153501
 Email: pd@cbs.dk  Priv: pda...@gmail.com









---
Marcos Amaris Gonzalez

__
R-help@r-project.org mailing list
https://stat.ethz.ch/mailman/listinfo/r-help
PLEASE do read the posting guide http://www.R-project.org/posting-guide.html
and provide commented, minimal, self-contained, reproducible code.


Re: [R] FIML with missing data in sem package

2011-12-01 Thread Rick Bilonick

On 12/01/2011 07:18 AM, John Fox wrote:

  To:r-help@r-project.org
  Subject: [R] FIML with missing data in sem package
  
You should check out the OpenMx R package. Just search for OpenMx and 
SEM. You can download from the web site. It does FIML and is an 
excellent SEM package.


Rick B .

__
R-help@r-project.org mailing list
https://stat.ethz.ch/mailman/listinfo/r-help
PLEASE do read the posting guide http://www.R-project.org/posting-guide.html
and provide commented, minimal, self-contained, reproducible code.


Re: [R] simplify source code

2011-12-01 Thread jim holtman
measurements[[colname]]

?'[['



On Thu, Dec 1, 2011 at 8:34 AM, Christof Kluß ckl...@email.uni-kiel.de wrote:
 Hi

 now I'd like to do

 for (colname in c('ColName1','ColName2','ColName3')) {
    dat - measurements$colname

 But that does not work, though I can write

 measurements$C1 (same as measurements$C1)
 (but different to measurements[C1]!)

 Can you give me a hint?

 greetings
 Christof



 Am 26-11-2011 23:30, schrieb Christof Kluß:

 Hi

 I would like to shorten

 mod1 - nls(ColName2 ~ ColName1, data = table, ...)
 mod2 - nls(ColName3 ~ ColName1, data = table, ...)
 mod3 - nls(ColName4 ~ ColName1, data = table, ...)
 ...

 is there something like

 cols = c(ColName2,ColName3,ColName4,...)

 for i in ...
 mod[i-1] - nls(ColName[i] ~ ColName1, data = table, ...)

 I am looking forward to help

 Christof


 __
 R-help@r-project.org mailing list
 https://stat.ethz.ch/mailman/listinfo/r-help
 PLEASE do read the posting guide http://www.R-project.org/posting-guide.html
 and provide commented, minimal, self-contained, reproducible code.



-- 
Jim Holtman
Data Munger Guru

What is the problem that you are trying to solve?
Tell me what you want to do, not how you want to do it.

__
R-help@r-project.org mailing list
https://stat.ethz.ch/mailman/listinfo/r-help
PLEASE do read the posting guide http://www.R-project.org/posting-guide.html
and provide commented, minimal, self-contained, reproducible code.


[R] hi all.regarding quantile regression results..

2011-12-01 Thread narendarreddy kalam
i know this is not about R.
After applying quantile regression with t=0.5,0.6  on the data set WBC(
Wisconsin Breast Cancer)with 678 observations and 9 independent
variables(inp1,inp2,...inp9) and 1 dependent variable(op)  i have got the 
following results for beta values.

when t=0.5(median regression) beta values   b1=0.002641,b2=0.045746,b3=0.
005282,b4=0.004397,b5=0.002641,b6=0.065807,b7=0.005282
,b8=0.031394,b9=0.004993 and intercept is -0.181388

and when t=0.6 beta values are
b1=0,b2=0.01,b3=0,b4=-0.002,b5=0.004,b6=0.,b7=0.002,b8=0,b9=0 and
intercept is -0.009 .

 sir,how to interpret the above beta coefficients and what do they mean
exactly??.
t=0.5 means are we considering first 50% of the total data?
t=0.6 means are we considering the first 60% of the total data?
 can we write a equation like

 
y=intercept+b1*inp11+b2*inp29+b3*inp3+b4*inp4+b5*inp5+b6*inp6+b7*inp7+b8*inp8+b9*inp9
as in Linear Regression
to calculate the predicted output of y or not??

 If we are taking  into  consideration 5 quantiles of data ,Does it  mean
that we are dividing data it into 5 parts??which variables i have to
consider if the data is to be divided into 5 parts?

And
  i got  5 equations for 5 quantiles, what exactly each equation represents?

  Can i write single equation for the data set as in mean regression by
combining the 5 equations of each quantile.
Please reply me.

Thanks In advance,

With regards

Kalam Naerndar Reddy
M.Tech(IT),
University of Hyderabad.

--
View this message in context: 
http://r.789695.n4.nabble.com/hi-all-regarding-quantile-regression-results-tp4128248p4128248.html
Sent from the R help mailing list archive at Nabble.com.

__
R-help@r-project.org mailing list
https://stat.ethz.ch/mailman/listinfo/r-help
PLEASE do read the posting guide http://www.R-project.org/posting-guide.html
and provide commented, minimal, self-contained, reproducible code.


Re: [R] References for book R In Action by Kabacoff

2011-12-01 Thread Tyler Rinker

In the ebook version there is a list of references (pp. 434-437).

 Date: Thu, 1 Dec 2011 10:48:45 +0100
 From: lig...@statistik.tu-dortmund.de
 To: ravi.k...@gmail.com
 CC: r-help@r-project.org
 Subject: Re: [R] References for book R In Action by Kabacoff
 
 On 01.12.2011 10:10, Ravi Kulkarni wrote:
  I know this is not really an R question - it is a query about a recent book
  on R (R In Action) by Robert Kabacoff, (Manning Publications 2011).
  There are many references to interesting topics in R in the book, BUT, I do
  not find a bibliography/list of references in the book!
  Does anybody know if there are errata for the book available some place?
 
 I'd ask the author!
 
 Uwe Ligges
 
 
 
  Thanks,
 Ravi
 
  --
  View this message in context: 
  http://r.789695.n4.nabble.com/References-for-book-R-In-Action-by-Kabacoff-tp4127625p4127625.html
  Sent from the R help mailing list archive at Nabble.com.
 
  __
  R-help@r-project.org mailing list
  https://stat.ethz.ch/mailman/listinfo/r-help
  PLEASE do read the posting guide http://www.R-project.org/posting-guide.html
  and provide commented, minimal, self-contained, reproducible code.
 
 __
 R-help@r-project.org mailing list
 https://stat.ethz.ch/mailman/listinfo/r-help
 PLEASE do read the posting guide http://www.R-project.org/posting-guide.html
 and provide commented, minimal, self-contained, reproducible code.
  
[[alternative HTML version deleted]]

__
R-help@r-project.org mailing list
https://stat.ethz.ch/mailman/listinfo/r-help
PLEASE do read the posting guide http://www.R-project.org/posting-guide.html
and provide commented, minimal, self-contained, reproducible code.


[R] Writing a function, want a string argument to define the name of the excel sheet to be called

2011-12-01 Thread AOLeary
My question is this: is there a way I can make one of the arguments of the
function be a string the user can enter, and then have that be the excel
filename? ie,

foo - function(x,y,NAME){

#make a matrix with x rows and y cols
M - matrix(nrow=x,ncol=y)
#write the matrix
write.table(M, file = result.csv,append=TRUE, sep = ,) 
}

I've had a look but I couldn't find help for this particular problem and
it's one I'd like to solve, so I can change make several excel files to
solve the analysis my actual function does.

Thank you very much,
Aodhán

--
View this message in context: 
http://r.789695.n4.nabble.com/Writing-a-function-want-a-string-argument-to-define-the-name-of-the-excel-sheet-to-be-called-tp4128384p4128384.html
Sent from the R help mailing list archive at Nabble.com.

__
R-help@r-project.org mailing list
https://stat.ethz.ch/mailman/listinfo/r-help
PLEASE do read the posting guide http://www.R-project.org/posting-guide.html
and provide commented, minimal, self-contained, reproducible code.


Re: [R] nipals in the chemometrics package in R

2011-12-01 Thread Bryan Hanson
zz dd:

If you want a typical score plot, say PC1 vs PC2, then plot the first and 
second columns of your score matrix (T) against each other and label the points 
with the row names (if available) or create a text label from where ever you 
have this information stored.  To get an unlabeled score plot, you could simply 
do:

plot(T[,1], T[,2])

If you are using the package chemometrics, you should take a look at the book 
that goes with it too: Varmuza and Filzmoser, Multivariate Statistical 
Analysis in Chemometrics.

If you need more guidance than this, you should probably give us more details 
about what you are doing.

Bryan

Prof. Bryan Hanson
Dept of Chemistry  Biochemistry
DePauw University
Greencastle IN 46135 USA
academic.depauw.edu/~hanson/deadpezsociety.html
github.com/bryanhanson
academic.depauw.edu/~hanson/UMP/Index.html

On Dec 1, 2011, at 6:51 AM, zz dd wrote:

 Hello
 i need some precision about nipals in the chemometrics package in R .
 
 When i use nipals in chemometrics i obtain T and P matrix.
 
 I really don't understand what to do with these two matrix to obtain
 the scores for every the component (like in spss fo example)
 
 Comp1Comp2   Comp3
 quest1 0,8434  0,54333   0,3466
 quest2 0,665   0,7655  0,433
 
 Thank you very much for your help
 (I know that X=TP+E)... But don't understand else
 
 __
 R-help@r-project.org mailing list
 https://stat.ethz.ch/mailman/listinfo/r-help
 PLEASE do read the posting guide http://www.R-project.org/posting-guide.html
 and provide commented, minimal, self-contained, reproducible code.

__
R-help@r-project.org mailing list
https://stat.ethz.ch/mailman/listinfo/r-help
PLEASE do read the posting guide http://www.R-project.org/posting-guide.html
and provide commented, minimal, self-contained, reproducible code.


Re: [R] Writing a function, want a string argument to define the name of the excel sheet to be called

2011-12-01 Thread Bryan Hanson
Sure, change your example as follows and then you can pass the name properly:

foo - function(x,y,NAME = filename.csv){

#make a matrix with x rows and y cols
M - matrix(nrow=x,ncol=y)
#write the matrix
write.table(M, file = NAME,append=TRUE, sep = ,) 
}

Bryan

Prof. Bryan Hanson
Dept of Chemistry  Biochemistry
DePauw University
Greencastle IN 46135 USA
academic.depauw.edu/~hanson/deadpezsociety.html
github.com/bryanhanson
academic.depauw.edu/~hanson/UMP/Index.html

On Dec 1, 2011, at 8:43 AM, AOLeary wrote:

 My question is this: is there a way I can make one of the arguments of the
 function be a string the user can enter, and then have that be the excel
 filename? ie,
 
 foo - function(x,y,NAME){
 
 #make a matrix with x rows and y cols
 M - matrix(nrow=x,ncol=y)
 #write the matrix
 write.table(M, file = result.csv,append=TRUE, sep = ,) 
 }
 
 I've had a look but I couldn't find help for this particular problem and
 it's one I'd like to solve, so I can change make several excel files to
 solve the analysis my actual function does.
 
 Thank you very much,
 Aodhán
 
 --
 View this message in context: 
 http://r.789695.n4.nabble.com/Writing-a-function-want-a-string-argument-to-define-the-name-of-the-excel-sheet-to-be-called-tp4128384p4128384.html
 Sent from the R help mailing list archive at Nabble.com.
 
 __
 R-help@r-project.org mailing list
 https://stat.ethz.ch/mailman/listinfo/r-help
 PLEASE do read the posting guide http://www.R-project.org/posting-guide.html
 and provide commented, minimal, self-contained, reproducible code.

__
R-help@r-project.org mailing list
https://stat.ethz.ch/mailman/listinfo/r-help
PLEASE do read the posting guide http://www.R-project.org/posting-guide.html
and provide commented, minimal, self-contained, reproducible code.


Re: [R] efficient ways to dynamically grow a dataframe

2011-12-01 Thread jim holtman
First, dataframes can be much slower than matrices, for example, if
you are changing/accessing values a lot.  I would suggest that you use
a matrix since is seems that all your values are numeric.  Allocate a
large empty matrix to start (hopefully as large as you need).  If you
exceed this, you have the option of 'rbind'ing more empty rows on and
continuing.  This might depend on how large your final matrix might be
(you did not state the boundary conditions).

On Thu, Dec 1, 2011 at 6:34 AM, Matteo Richiardi
matteo.richia...@unito.it wrote:
 Hi,
 I'm trying to write a small microsimulation in R: that is, I have a
 dataframe with info on N individuals for the base-year and I have to
 grow it dynamically for T periods:

 df = data.frame(
  id = 1:N,
  x =
 )

 The most straightforward way to solve the problem that came to my mind
 is to create for every period a new dataframe:

 for(t in 1:T){
  for(i in 1:N){
  row = data.frame(
   id = i,
   t = t,
   x = ...
   )
   df = rbind(df,row)
  }
 }

 This is very inefficient and my pc gets immediately stucked as N is
 raised above some thousands.
 As an alternative, I created an empty dataframe for all the projected
 periods, and then filled it:

 df1 = data.frame(
  id = rep(1:N,T),
  t = rep(1:T, each = N),
  x = rep(NA,N*T)
 )

 for(t in 1:T){
  for(i in 1:N){
  x = ...
  df1[df1$id==i  df1$t==t,x] = x
  }
 }
 df = rbind(df,df1)

 This is also too slow, and my PC gets stucked. I don't want to go for
 a matrix, because I'd loose the column names and everything will
 become too much error-prone.
 Any suggestions on how to do it?
 Thanks in advance,
 Matteo




 --
 Matteo Richiardi
 University of Turin
 Faculty of Law
 Department of Economics Cognetti De Martiis
 via Po 53, 10124 Torino
 Email: matteo.richia...@unito.it
 Tel. +39 011 670 3870
 Web page: http://www.personalweb.unito.it/matteo.richiardi/

 __
 R-help@r-project.org mailing list
 https://stat.ethz.ch/mailman/listinfo/r-help
 PLEASE do read the posting guide http://www.R-project.org/posting-guide.html
 and provide commented, minimal, self-contained, reproducible code.



-- 
Jim Holtman
Data Munger Guru

What is the problem that you are trying to solve?
Tell me what you want to do, not how you want to do it.

__
R-help@r-project.org mailing list
https://stat.ethz.ch/mailman/listinfo/r-help
PLEASE do read the posting guide http://www.R-project.org/posting-guide.html
and provide commented, minimal, self-contained, reproducible code.


Re: [R] ordinal data simulation

2011-12-01 Thread David Winsemius


On Dec 1, 2011, at 2:06 AM, yuying shi wrote:


Dear all,
  I am doing an ordinal data simulation. I have a question  
regarding the cut off values between categories. Suppose I have  
three categories, if I do regression, there should be two cut off  
values. I find some simulation code for the ordinal data. However,  
they usually only generate a uniform random number and compare the  
probability with the random number.


That's not a good description of what they are doing.

Could any one be so kind to tell me the rational behind it. I don't  
know why we need a random number rather than  just  two fixed cut  
off values.


One way to generate a random categorical variable constrained to three  
values would just be to take a uniform variate on the interval [0,1]  
and classify it based on whether which of the three segments it is in  
for 0  a  b  1. This really is not an appropriate question for  
rhelp so you are advised to submit further such questions somewhere  
they are solicited such as statsexchange.com.


--

David Winsemius, MD
West Hartford, CT

__
R-help@r-project.org mailing list
https://stat.ethz.ch/mailman/listinfo/r-help
PLEASE do read the posting guide http://www.R-project.org/posting-guide.html
and provide commented, minimal, self-contained, reproducible code.


Re: [R] Raster manipulation in R

2011-12-01 Thread jon . skoien

Hi,

Alternatively you could have a look at the function terrain in the 
raster package, it can calculate the slope for you using different 
algorithms, not sure if the one below is included though.


For typical spatial requests like this, you could also use the 
mailing-list r-sig-geo.


Cheers,
Jon

On 01-Dec-11 14:27, Jean V Adams wrote:

jjcabrera20 wrote on 12/01/2011 04:02:47 AM:


Hello everyone in the forum
For introducing myself I would say I have a basic knowledge of R.
Now I am intended to implement a flood algorithm using R. I have some

MatLab

experience doing these, and as an example to explain more or less what I
want, I have a m code to calculate the slope from a Digital elevation

model:

slope=zeros(row,col);
for i=2:row-1
 for j=2:col-1
dzdx=(raster(i,j+1)-raster(i,j-1))/(2*res);
dzdy=(raster(i+1,j)-raster(i-1,j))/(2*res);
slope(i,j)=sqrt(dzdx^2+dzdy^2);
 end
end;

The question is to know how to do the similar procedure on R. All
suggestions are welcome
Thanks

All best,

Jorge
PD:I am using R on windows system 64 bits


If I am interpreting your code correctly (I don't use MatLab myself),
something like this should give you the same result in R:

# example matrix of heights
raster- matrix(runif(20, 10, 30), nrow=4, ncol=5)

# example resolution
res- 8.5

# dimensions of matrix
drast- dim(raster)

# for every non-boundary point in the matrix,
# calculate the distances between its adjacent columns (dzdx) and rows
(dzdy)
dzdx- raster[2:(drast[1]-1), 3:drast[2]] - raster[2:(drast[1]-1),
1:(drast[2]-2)]
dzdy- raster[3:drast[1], 2:(drast[2]-1)] - raster[1:(drast[1]-2),
2:(drast[2]-1)]

# calculate the slope from these distances and the resolution
slope- sqrt(dzdx^2 + dzdy^2) / (2*res)


Jean
[[alternative HTML version deleted]]

__
R-help@r-project.org mailing list
https://stat.ethz.ch/mailman/listinfo/r-help
PLEASE do read the posting guide http://www.R-project.org/posting-guide.html
and provide commented, minimal, self-contained, reproducible code.



--
Jon Olav Skøien
Joint Research Centre - European Commission
Institute for Environment and Sustainability (IES)
Global Environment Monitoring Unit

Via Fermi 2749, TP 440,  I-21027 Ispra (VA), ITALY

jon.sko...@jrc.ec.europa.eu
Tel:  +39 0332 789206

Disclaimer: Views expressed in this email are those of the individual and do 
not necessarily represent official views of the European Commission.

__
R-help@r-project.org mailing list
https://stat.ethz.ch/mailman/listinfo/r-help
PLEASE do read the posting guide http://www.R-project.org/posting-guide.html
and provide commented, minimal, self-contained, reproducible code.


Re: [R] R logo in eps formt

2011-12-01 Thread Matt Shotwell
See this earlier post for SVG logos:

http://tolstoy.newcastle.edu.au/R/e12/devel/10/10/0112.html

Using Image Magick, do something like 

convert logo.svg logo.eps


On Thu, 2011-12-01 at 10:56 +0700, Ben Madin wrote:
 G'day all,
 
 Sorry if this message has been posted before, but searching for R is always 
 difficult...
 
 I was hoping for a copy of the logo in eps format? Can I do this from R, or 
 is one available for download?
 
 cheers
 
 Ben
 __
 R-help@r-project.org mailing list
 https://stat.ethz.ch/mailman/listinfo/r-help
 PLEASE do read the posting guide http://www.R-project.org/posting-guide.html
 and provide commented, minimal, self-contained, reproducible code.

__
R-help@r-project.org mailing list
https://stat.ethz.ch/mailman/listinfo/r-help
PLEASE do read the posting guide http://www.R-project.org/posting-guide.html
and provide commented, minimal, self-contained, reproducible code.


Re: [R] FIML with missing data in sem package

2011-12-01 Thread Dustin Fife
Good idea. I'll give it a try. Thanks!

On Thu, Dec 1, 2011 at 6:18 AM, John Fox j...@mcmaster.ca wrote:

 Dear Dustin,

  -Original Message-
  From: r-help-boun...@r-project.org [mailto:r-help-bounces@r-
  project.org] On Behalf Of Dustin Fife
  Sent: November-30-11 9:51 PM
  To: r-help@r-project.org
  Subject: [R] FIML with missing data in sem package
 
  Is there a way to use full information maximum likelihood (FIML) to
  estimate missing data in the sem package? For example, suppose I have a
  dataset with complete information on X1-X3, but missing data (MAR) on
  X4.
  Is there a way to use FIML in this case? I know lavaan and openmx allow

 No, the sem package doesn't handle missing data. You could, however, use
 another package, such as mice, for multiple imputation of the missing data.

 Best,
  John

 
 John Fox
 Senator William McMaster
  Professor of Social Statistics
 Department of Sociology
 McMaster University
 Hamilton, Ontario, Canada
 http://socserv.mcmaster.ca/jfox


  you to do it, but I couldn't find anything in the documentation for the
  sem package. Thanks!
 
  --
  Dustin Fife
  Graduate Student
  Quantitative Psychology
  University of Oklahoma
 
[[alternative HTML version deleted]]
 
  __
  R-help@r-project.org mailing list
  https://stat.ethz.ch/mailman/listinfo/r-help
  PLEASE do read the posting guide http://www.R-project.org/posting-
  guide.html
  and provide commented, minimal, self-contained, reproducible code.




-- 
Dustin Fife
Graduate Student
Quantitative Psychology
University of Oklahoma

[[alternative HTML version deleted]]

__
R-help@r-project.org mailing list
https://stat.ethz.ch/mailman/listinfo/r-help
PLEASE do read the posting guide http://www.R-project.org/posting-guide.html
and provide commented, minimal, self-contained, reproducible code.


Re: [R] how to get inflection point in binomial glm

2011-12-01 Thread Rubén Roa
Assuming a logistic model, for each group solve for d at Y=0, or solve for d at 
p=0.5, where d is your continuous predictor, Y is the logit score, and p is the 
probability of success in the binomial model. After that you can get the 
standard error of the inflection point by Taylor series (aka delta method).

Another idea is to re-parameterize the logistic model to include explicitly the 
inflection point, thus you'll get its estimate and standard error directly as a 
result of the fit.
For example, disregarding the g factor predictor (or for each group), a 
logistic model such as
P(d) = 1/(1+exp(log(1/19)(d-d50)/(d95-d50))
includes the inflection point directly (d50) and is a re-parameterized version 
of the usual logistic model
P(d) =1/(1+exp(b0+b1*d))
where parameters b0 and b1 have been replaced by d95 and d50, the predictor at 
50% probability (inflection point), and the predictor at 95% probability, 
respectively.

HTH

Rubén

-Mensaje original-
De: r-help-boun...@r-project.org [mailto:r-help-boun...@r-project.org] En 
nombre de René Mayer
Enviado el: jueves, 01 de diciembre de 2011 14:25
Para: r-help@r-project.org
Asunto: [R] how to get inflection point in binomial glm

Dear All,

I have a binomial response with one continuous predictor (d) and one factor (g) 
(8 levels dummy-coded).

glm(resp~d*g, data, family=binomial)

Y=b0+b1*X1+b2*X2 ... b7*X7

how can I get the inflection point per group, e.g., P(d)=.5

I would be grateful for any help.

Thanks in advance,
René

__
R-help@r-project.org mailing list
https://stat.ethz.ch/mailman/listinfo/r-help
PLEASE do read the posting guide http://www.R-project.org/posting-guide.html
and provide commented, minimal, self-contained, reproducible code.

__
R-help@r-project.org mailing list
https://stat.ethz.ch/mailman/listinfo/r-help
PLEASE do read the posting guide http://www.R-project.org/posting-guide.html
and provide commented, minimal, self-contained, reproducible code.


Re: [R] x, y for point of intersection

2011-12-01 Thread Monica Pisica

Hi everybody,

Thanks for checking my code, Hans, it help to see where my initial mistake was. 
I am sorry i assumed that there was a minimization problem. 

In short i had 2 wavy lines (left and right) that didn't intersect and lots 
of straight parallel lines that intersect the first 2 lines. I wanted to solve 
the intersection for the left and right line simultaneously and put everything 
in a big loop. And there the mistake was.

Meanwhile i was able to use Don suggestion with no problem and no loop.

Thanks again,

Monica

 Date: Thu, 1 Dec 2011 12:13:49 +0100
 Subject: Re: [R] x, y for point of intersection
 From: hwborch...@googlemail.com
 To: r-help@r-project.org
 CC: macque...@llnl.gov; pisican...@hotmail.com; dwinsem...@comcast.net; 
 ted.hard...@wlandres.net
 
 Monica has sent me some data and code for taking a quick look. As it
 turned out, there was a simple programming error on her side. The
 segm_distance() function in package 'pracma' is working correctly. And
 there is no minimization procedure in here, it simply solves some
 equations from plane geometry.
 
 Maybe the function suggested by Don is faster, I haven't checked that.
 
 Regards,  Hans Werner
 
 
 2011/11/29 Monica Pisica pisican...@hotmail.com
 
  Hi again,
 
  Working with my real data and not with the little example i sent to the 
  list i discovered that segm_distance function from package pracma does 
  not converge to 0 in all cases, even if i increase the number of iteration 
  to 10,000 for example. It seems that it depends on the initialization 
  point - most like a minimization function.
 
  So my thanks  go to Don who's suggestion works for the real data as well 
  without any problems - so far ;-) He suggested to use the function 
  crossing.psp from package spatstat.
 
  Thanks again to all who have answered and helped to solve my problem. I 
  certainly learned few new things.
 
  Monica
 
 
 
 
 
   From: macque...@llnl.gov
   To: pisican...@hotmail.com
   CC: r-help@r-project.org
   Date: Wed, 23 Nov 2011 14:03:42 -0800
 
   Subject: Re: [R] x, y for point of intersection
  
   The function crossing.psp() in the spatstat package might be of use.
   Here's an excerpt from its help page:
  
   crossing.psp package:spatstat R Documentation
   Crossing Points of Two Line Segment PatternsDescription:
   Finds any crossing points between two line segment patterns.
   Usage:
   crossing.psp(A,B)
   -Don
  
  
   --
   Don MacQueen
  
   Lawrence Livermore National Laboratory
   7000 East Ave., L-627
   Livermore, CA 94550
   925-423-1062
  
 
  
[[alternative HTML version deleted]]

__
R-help@r-project.org mailing list
https://stat.ethz.ch/mailman/listinfo/r-help
PLEASE do read the posting guide http://www.R-project.org/posting-guide.html
and provide commented, minimal, self-contained, reproducible code.


Re: [R] hi all.regarding quantile regression results..

2011-12-01 Thread R. Michael Weylandt michael.weyla...@gmail.com
This really isn't the appropriate forum for most of your questions: I'd suggest 
you work through the Wikipedia article on quantiles regression and direct 
follow up to stats.stackexchange.com. 

As to the R question: use the predict() function. 

Michael

On Dec 1, 2011, at 8:12 AM, narendarreddy kalam narendarcse...@gmail.com 
wrote:

 i know this is not about R.
 After applying quantile regression with t=0.5,0.6  on the data set WBC(
 Wisconsin Breast Cancer)with 678 observations and 9 independent
 variables(inp1,inp2,...inp9) and 1 dependent variable(op)  i have got the 
 following results for beta values.
 
 when t=0.5(median regression) beta values   b1=0.002641,b2=0.045746,b3=0.
 005282,b4=0.004397,b5=0.002641,b6=0.065807,b7=0.005282
 ,b8=0.031394,b9=0.004993 and intercept is -0.181388
 
 and when t=0.6 beta values are
 b1=0,b2=0.01,b3=0,b4=-0.002,b5=0.004,b6=0.,b7=0.002,b8=0,b9=0 and
 intercept is -0.009 .
 
 sir,how to interpret the above beta coefficients and what do they mean
 exactly??.
 t=0.5 means are we considering first 50% of the total data?
 t=0.6 means are we considering the first 60% of the total data?
 can we write a equation like
 
 
 y=intercept+b1*inp11+b2*inp29+b3*inp3+b4*inp4+b5*inp5+b6*inp6+b7*inp7+b8*inp8+b9*inp9
 as in Linear Regression
 to calculate the predicted output of y or not??
 
 If we are taking  into  consideration 5 quantiles of data ,Does it  mean
 that we are dividing data it into 5 parts??which variables i have to
 consider if the data is to be divided into 5 parts?
 
 And
  i got  5 equations for 5 quantiles, what exactly each equation represents?   
  
  Can i write single equation for the data set as in mean regression by
 combining the 5 equations of each quantile.
 Please reply me.
 
 Thanks In advance,
 
With regards
 
 Kalam Naerndar Reddy
M.Tech(IT),
 University of Hyderabad.
 
 --
 View this message in context: 
 http://r.789695.n4.nabble.com/hi-all-regarding-quantile-regression-results-tp4128248p4128248.html
 Sent from the R help mailing list archive at Nabble.com.
 
 __
 R-help@r-project.org mailing list
 https://stat.ethz.ch/mailman/listinfo/r-help
 PLEASE do read the posting guide http://www.R-project.org/posting-guide.html
 and provide commented, minimal, self-contained, reproducible code.

__
R-help@r-project.org mailing list
https://stat.ethz.ch/mailman/listinfo/r-help
PLEASE do read the posting guide http://www.R-project.org/posting-guide.html
and provide commented, minimal, self-contained, reproducible code.


Re: [R] efficient ways to dynamically grow a dataframe

2011-12-01 Thread R. Michael Weylandt
I'd also suggest you read circle 2 of the R inferno (just google it)
which has some helpful tips on how to deal with these sorts of
problems.

Also, did you know that matrices can have column names and that
rbind() preserves them? E.g.,

m - matrix(1:6, 3); colnames(m) - letters[1:2]

print(m)

print(rbind(m, c(10, 11)))

Michael

On Thu, Dec 1, 2011 at 9:02 AM, jim holtman jholt...@gmail.com wrote:
 First, dataframes can be much slower than matrices, for example, if
 you are changing/accessing values a lot.  I would suggest that you use
 a matrix since is seems that all your values are numeric.  Allocate a
 large empty matrix to start (hopefully as large as you need).  If you
 exceed this, you have the option of 'rbind'ing more empty rows on and
 continuing.  This might depend on how large your final matrix might be
 (you did not state the boundary conditions).

 On Thu, Dec 1, 2011 at 6:34 AM, Matteo Richiardi
 matteo.richia...@unito.it wrote:
 Hi,
 I'm trying to write a small microsimulation in R: that is, I have a
 dataframe with info on N individuals for the base-year and I have to
 grow it dynamically for T periods:

 df = data.frame(
  id = 1:N,
  x =
 )

 The most straightforward way to solve the problem that came to my mind
 is to create for every period a new dataframe:

 for(t in 1:T){
  for(i in 1:N){
  row = data.frame(
   id = i,
   t = t,
   x = ...
   )
   df = rbind(df,row)
  }
 }

 This is very inefficient and my pc gets immediately stucked as N is
 raised above some thousands.
 As an alternative, I created an empty dataframe for all the projected
 periods, and then filled it:

 df1 = data.frame(
  id = rep(1:N,T),
  t = rep(1:T, each = N),
  x = rep(NA,N*T)
 )

 for(t in 1:T){
  for(i in 1:N){
  x = ...
  df1[df1$id==i  df1$t==t,x] = x
  }
 }
 df = rbind(df,df1)

 This is also too slow, and my PC gets stucked. I don't want to go for
 a matrix, because I'd loose the column names and everything will
 become too much error-prone.
 Any suggestions on how to do it?
 Thanks in advance,
 Matteo




 --
 Matteo Richiardi
 University of Turin
 Faculty of Law
 Department of Economics Cognetti De Martiis
 via Po 53, 10124 Torino
 Email: matteo.richia...@unito.it
 Tel. +39 011 670 3870
 Web page: http://www.personalweb.unito.it/matteo.richiardi/

 __
 R-help@r-project.org mailing list
 https://stat.ethz.ch/mailman/listinfo/r-help
 PLEASE do read the posting guide http://www.R-project.org/posting-guide.html
 and provide commented, minimal, self-contained, reproducible code.



 --
 Jim Holtman
 Data Munger Guru

 What is the problem that you are trying to solve?
 Tell me what you want to do, not how you want to do it.

 __
 R-help@r-project.org mailing list
 https://stat.ethz.ch/mailman/listinfo/r-help
 PLEASE do read the posting guide http://www.R-project.org/posting-guide.html
 and provide commented, minimal, self-contained, reproducible code.

__
R-help@r-project.org mailing list
https://stat.ethz.ch/mailman/listinfo/r-help
PLEASE do read the posting guide http://www.R-project.org/posting-guide.html
and provide commented, minimal, self-contained, reproducible code.


[R] round to specific intervals

2011-12-01 Thread Michael Kao

Dear R users/helpers,

I am wondering is there an existing function in which you can round 
numbers to a set of values. I know you can use 5 * round(x/5) for 
rounding to the nearest 5 or so, but what if the interval size is not 
constant.


For example:
## Not run
test - rnorm(100)
round(test, c(1, 5, 10, 20, 50))

so that the test is rounded to the closest value in the vector.

Thanks for the help.

Cheers,

__
R-help@r-project.org mailing list
https://stat.ethz.ch/mailman/listinfo/r-help
PLEASE do read the posting guide http://www.R-project.org/posting-guide.html
and provide commented, minimal, self-contained, reproducible code.


Re: [R] Nomogram with stratified cph in rms package, how to get failure probability

2011-12-01 Thread Frank Harrell
Hazard() is not implemented except for parametric survival models.

You are not calling nomogram() correctly; in rms the plotting step is
separated from the nomogram computations.

To plot cumulative event rates do something like:

mort10 - function(lp) 1 - surv(10,lp)

and tell nomogram about mort10 using fun=.

Frank



min wrote
 
 Hello,
 
 I am using Dr. Harrell's rms package to make a nomogram. I was able to
 make a beautiful one. However, I want to change 5-year survival
 probability to 5-year failure probability.
 
 I couldn’t get hazard rate from Hazard(f1) because I used cph for the
 model.
 
 Here is my code:
 
  library(rms)
 f1 - cph(Surv(retime,dfs) ~
 age+her2+t_stage+n_stage+er+grade+cytcyt+Cyt_PCDK2  , data=data11,
 surv=T, x=T, y=T, time.inc=5)
 
 surv- Survival(f1)
 haz- Hazard(f1)
 Here is the Error in UseMethod(Hazard) :
   no applicable method for 'Hazard' applied to an object of class
 c('cph', 'Design', 'coxph')
 
 
 surv10 - function(lp) surv(10,lp)
 surv5 - function(lp) surv(5,lp)
 quant - Quantile(f1)
 
 at.surv - c(0.1, 0.3, 0.5,  0.7, 0.9)
 
 at.med1-c(2,3,4, 5,6,7,8, 10,15,20,25, 30)
 
 
 par(cex=0.8)
 nom- nomogram(f1, conf.int=F,
 fun=list(surv5, surv10), funlabel=c('5-Year Survival Probability',
 '10-Year Survival Probability' ),  lp=F,
 fun.at=c(at.surv, at.surv),label.every=1, force.label=FALSE, cex.axis=0.8,
 verbose=TRUE, cex.var=0.8)
 
 How can I show failure probability instead of survival probability? 
 
 I would very much appreciate any assistance in this matter.
 Thank you Very much.
 


-
Frank Harrell
Department of Biostatistics, Vanderbilt University
--
View this message in context: 
http://r.789695.n4.nabble.com/Nomogram-with-stratified-cph-in-rms-package-how-to-get-failure-probability-tp4123249p4129173.html
Sent from the R help mailing list archive at Nabble.com.

__
R-help@r-project.org mailing list
https://stat.ethz.ch/mailman/listinfo/r-help
PLEASE do read the posting guide http://www.R-project.org/posting-guide.html
and provide commented, minimal, self-contained, reproducible code.


Re: [R] round to specific intervals

2011-12-01 Thread Bert Gunter
?findInterval
.. would get you the endpoints and then you could determine which is nearer.

Of course in your example, everything would get rounded to 1.

-- Bert

On Thu, Dec 1, 2011 at 7:55 AM, Michael Kao mkao006rm...@gmail.com wrote:
 Dear R users/helpers,

 I am wondering is there an existing function in which you can round numbers
 to a set of values. I know you can use 5 * round(x/5) for rounding to the
 nearest 5 or so, but what if the interval size is not constant.

 For example:
 ## Not run
 test - rnorm(100)
 round(test, c(1, 5, 10, 20, 50))

 so that the test is rounded to the closest value in the vector.

 Thanks for the help.

 Cheers,

 __
 R-help@r-project.org mailing list
 https://stat.ethz.ch/mailman/listinfo/r-help
 PLEASE do read the posting guide http://www.R-project.org/posting-guide.html
 and provide commented, minimal, self-contained, reproducible code.



-- 

Bert Gunter
Genentech Nonclinical Biostatistics

Internal Contact Info:
Phone: 467-7374
Website:
http://pharmadevelopment.roche.com/index/pdb/pdb-functional-groups/pdb-biostatistics/pdb-ncb-home.htm

__
R-help@r-project.org mailing list
https://stat.ethz.ch/mailman/listinfo/r-help
PLEASE do read the posting guide http://www.R-project.org/posting-guide.html
and provide commented, minimal, self-contained, reproducible code.


Re: [R] Writing a function, want a string argument to define the name of the excel sheet to be called

2011-12-01 Thread AOLeary
Thank you very much, that does exactly what I want it to! :)

Aodhán

--
View this message in context: 
http://r.789695.n4.nabble.com/Writing-a-function-want-a-string-argument-to-define-the-name-of-the-excel-sheet-to-be-called-tp4128384p4128495.html
Sent from the R help mailing list archive at Nabble.com.

__
R-help@r-project.org mailing list
https://stat.ethz.ch/mailman/listinfo/r-help
PLEASE do read the posting guide http://www.R-project.org/posting-guide.html
and provide commented, minimal, self-contained, reproducible code.


[R] R, PostgresSQL and poor performance

2011-12-01 Thread Berry, David I.
Hi List

Apologies if this isn't the correct place for this query (I've tried a search 
of the mail archives but not had much joy).

I'm running R (2.14.0)  on a Mac (OSX v 10.5.8, 2.66GHz, 4GB memory) and am 
having a few performance issues with reading data in from a Postres database 
(using RPostgreSQL). My query / code are as below

# -
library('RPostgreSQL')

drv - dbDriver(PostgreSQL)

dbh - dbConnect(drv,user=…,password=…,dbname=…,host=…)

sql - select id, date, lon, lat, date_trunc('day' , date) as jday, 
extract('hour' from date) as hour, extract('year' from date) as year from 
observations where pt = 6 and date = '1990-01-01' and date  '1995-01-01' and 
lon  180 and lon  290 and lat  -30 and lat  30 and sst is not null

dataIn - dbGetQuery(dbh,sql)
# -


All variables are reals other than id which is varchar(10) and date which is a 
timestamp, approximately 1.5 million rows are returned by the query and it 
takes order 10 second to execute using psql (the command line client for 
Postgres) and a similar time using pgAdmin 3. In R it takes several minutes to 
run and I'm unsure where the bottleneck is occurring.  I don't believe it's in 
the database access as the query disappears from the server status (of pgAdmin) 
quite quickly and where it only takes ~10 seconds from psql.  I've only been 
using R for the last month or two so any tips / advice on how to speed up the 
query or how to track down the source of the poor performance would be 
appreciated.

For info, I get similar poor performance running R (2.14.0) on a linux 
workstation (Linux 2.6.16.60-0.91.1-smp x86_64, 42GB memory, 2 6 core 
processors) running SUSE.

Thanks in advance

Dave Berry.
-- 
This message (and any attachments) is for the recipient ...{{dropped:8}}

__
R-help@r-project.org mailing list
https://stat.ethz.ch/mailman/listinfo/r-help
PLEASE do read the posting guide http://www.R-project.org/posting-guide.html
and provide commented, minimal, self-contained, reproducible code.


[R] How to do Hotelling's t2 test?

2011-12-01 Thread jpquinn91
Hi, I want to do a 2 sample hotelling's test but i can't figure out how. When
i type T2.test it says there is no such test and when i tried library(rrcov)
it says there is no such program. Cheers.

--
View this message in context: 
http://r.789695.n4.nabble.com/How-to-do-Hotelling-s-t2-test-tp4128748p4128748.html
Sent from the R help mailing list archive at Nabble.com.

__
R-help@r-project.org mailing list
https://stat.ethz.ch/mailman/listinfo/r-help
PLEASE do read the posting guide http://www.R-project.org/posting-guide.html
and provide commented, minimal, self-contained, reproducible code.


Re: [R] Resampling with replacement on a binary (0, 1) dataset to get Cis

2011-12-01 Thread lincoln
Thanks.

So, suppose for one specific year (first year over 10) the percentage of
successes deriving from 100 trials with 38 successes (and 62 failures), its
value would be 38/100=0.38.
I could calculate its confidence intervals this way:
 success-38
 total-100
 prop.test(success,total,p=0.5,alternative=two.sided)

1-sample proportions test with continuity
correction

data:  success out of total, null probability 0.5 
X-squared = 5.29, df = 1, p-value = 0.02145
alternative hypothesis: true p is not equal to 0.5 
95 percent confidence interval:
 0.2863947 0.4829411 
sample estimates:
   p 
0.38 

So it would be var$1=0.38 , CI=0.286-0.483

Is it correct?

--
View this message in context: 
http://r.789695.n4.nabble.com/Resampling-with-replacement-on-a-binary-0-1-variable-to-get-CIs-tp4127990p4129048.html
Sent from the R help mailing list archive at Nabble.com.

__
R-help@r-project.org mailing list
https://stat.ethz.ch/mailman/listinfo/r-help
PLEASE do read the posting guide http://www.R-project.org/posting-guide.html
and provide commented, minimal, self-contained, reproducible code.


Re: [R] How to speed up int2bin conversion?

2011-12-01 Thread Jonas Jägermeyr

On 12/01/2011 02:46 PM, jim holtman wrote:

If we assume that you are just convert 8 bits, here is one way with a
table lookup.  If more than 8 bits, just partition the data and
repeat.  This sets up a mapping table one time for the lookup.  Does
1M in 0.3 seconds on my computer; is this fast enough?


# initialize a matrix with 8 bit binary values
# one time
require(bitops)
b2c- character(256)
for (i in 0:255){

+ b2c[i + 1]- sprintf(%1d%1d%1d%1d%1d%1d%1d%1d
+ , bitAnd(i, 0x80) != 0
+ , bitAnd(i, 0x40) != 0
+ , bitAnd(i, 0x20) != 0
+ , bitAnd(i, 0x10) != 0
+ , bitAnd(i, 0x8) != 0
+ , bitAnd(i, 0x4) != 0
+ , bitAnd(i, 0x2) != 0
+ , bitAnd(i, 0x1) != 0
+ )
+ }

# create vector with 1M values
x- as.integer(1:1e6)

int2bin- function(val)

+ {
+ b2c[bitAnd(val, 0xff) + 1]
+ }

system.time(int2bin(x))

user  system elapsed
0.310.000.32

On Thu, Dec 1, 2011 at 7:14 AM, Jonas Jägermeyrjonas...@pik-potsdam.de  wrote:

Dear R-help members,

I'm processing a large amount of MODIS data where quality assessment
information is stored as an integer value for each pixel. I have to
converted this number to an 8 digit binary flag to get access to the
stored quality code (e.g. in2bin(165,8) = 1 0 1 0 0 1 0 1).

Unfortunately, I did not manage to find a package providing a fast
function to do so. I need to run this on millions of pixels and thus
wrote the following function.

int2bin- function(x,ndigits){
 base- array(NA, dim=c(length(x), ndigits))
 for(q in 1:ndigits){
   base[, ndigits-q+1]- (x %% 2)
   x- (x %/% 2)
   }
 bin- apply(base,1,paste,collapse=)
 return(bin)
}

Since it is still slow, I have to find a way to express this more
elegantly. I'd really appreciate any help.

Thanking you, with best regards

Jonas Jägermeyr

Potsdam Institute for Climate Impact Research
Research Domain II

[[alternative HTML version deleted]]


__
R-help@r-project.org mailing list
https://stat.ethz.ch/mailman/listinfo/r-help
PLEASE do read the posting guide http://www.R-project.org/posting-guide.html
and provide commented, minimal, self-contained, reproducible code.





Great Jim! Thanks a lot. New way of thinking, appreciated.
I also need 16 and 32 bit flags and hence I just added those two lines:

int2bin16- function(val) {
paste(int2bin(val%/%256),int2bin(val),sep=)
}

int2bin32- function(val) {

paste(int2bin(val%/%256^3),int2bin(val%/%256^2),int2bin(val%/%256),int2bin(val),sep=)

}

Best,
Jonas Jägermeyr

Potsdam Institute for Climate Impact Research
Research Domain II

__
R-help@r-project.org mailing list
https://stat.ethz.ch/mailman/listinfo/r-help
PLEASE do read the posting guide http://www.R-project.org/posting-guide.html
and provide commented, minimal, self-contained, reproducible code.


Re: [R] FIML with missing data in sem package

2011-12-01 Thread Dustin Fife

 What is your goal?  I have used and like mice pretty well, but using
 mice + sem to try to address missingness seems like more work than
 using FIML in OpenMx or lavaan to try to address it.  Is there a
 reason you want to use the sem package or a reason you do not want to
 use the others?

 I tried lavaan and couldn't get it to work. I noticed it was in beta so I
figured it wasn't a good idea to use for simulations I intend to publish
(especially when I can't get them to work properly). Either way, whether I
use OpenMX or lavaan, it will require me to learn a new syntax, and I guess
I'm just lazy. I'm comfortable with sem.



 On Thu, Dec 1, 2011 at 6:27 AM, Dustin Fife fife.dus...@gmail.com wrote:
  Good idea. I'll give it a try. Thanks!
 
  On Thu, Dec 1, 2011 at 6:18 AM, John Fox j...@mcmaster.ca wrote:
 
  Dear Dustin,
 
   -Original Message-
   From: r-help-boun...@r-project.org [mailto:r-help-bounces@r-
   project.org] On Behalf Of Dustin Fife
   Sent: November-30-11 9:51 PM
   To: r-help@r-project.org
   Subject: [R] FIML with missing data in sem package
  
   Is there a way to use full information maximum likelihood (FIML) to
   estimate missing data in the sem package? For example, suppose I have
 a
   dataset with complete information on X1-X3, but missing data (MAR) on
   X4.
   Is there a way to use FIML in this case? I know lavaan and openmx
 allow
 
  No, the sem package doesn't handle missing data. You could, however, use
  another package, such as mice, for multiple imputation of the missing
 data.
 
  Best,
   John
 
  
  John Fox
  Senator William McMaster
   Professor of Social Statistics
  Department of Sociology
  McMaster University
  Hamilton, Ontario, Canada
  http://socserv.mcmaster.ca/jfox
 
 
   you to do it, but I couldn't find anything in the documentation for
 the
   sem package. Thanks!
  
   --
   Dustin Fife
   Graduate Student
   Quantitative Psychology
   University of Oklahoma
  
 [[alternative HTML version deleted]]
  
   __
   R-help@r-project.org mailing list
   https://stat.ethz.ch/mailman/listinfo/r-help
   PLEASE do read the posting guide http://www.R-project.org/posting-
   guide.html
   and provide commented, minimal, self-contained, reproducible code.
 
 
 
 
  --
  Dustin Fife
  Graduate Student
  Quantitative Psychology
  University of Oklahoma
 
 [[alternative HTML version deleted]]
 
  __
  R-help@r-project.org mailing list
  https://stat.ethz.ch/mailman/listinfo/r-help
  PLEASE do read the posting guide
 http://www.R-project.org/posting-guide.html
  and provide commented, minimal, self-contained, reproducible code.



 --
 Joshua Wiley
 Ph.D. Student, Health Psychology
 Programmer Analyst II, ATS Statistical Consulting Group
 University of California, Los Angeles
 https://joshuawiley.com/




-- 
Dustin Fife
Graduate Student
Quantitative Psychology
University of Oklahoma

[[alternative HTML version deleted]]

__
R-help@r-project.org mailing list
https://stat.ethz.ch/mailman/listinfo/r-help
PLEASE do read the posting guide http://www.R-project.org/posting-guide.html
and provide commented, minimal, self-contained, reproducible code.


Re: [R] Nomogram with stratified cph in rms package, how to get failure probability

2011-12-01 Thread min
Got it.

Thank you so much for the help!

Min Yi  MD, PhD
Department of Surgical Oncology - Unit 1484
1400 Pressler Street, #FCT17.6061
University of Texas M.D. Anderson Cancer Center
P.O. Box 301402
Houston, TX  77230-1402
Phone: 713-563-1874
Fax: 713-792-4689

From: Frank Harrell [via R] [mailto:ml-node+s789695n4129173...@n4.nabble.com]
Sent: Thursday, December 01, 2011 10:15 AM
To: Yi,Min
Subject: Re: Nomogram with stratified cph in rms package, how to get failure 
probability

Hazard() is not implemented except for parametric survival models.

You are not calling nomogram() correctly; in rms the plotting step is separated 
from the nomogram computations.

To plot cumulative event rates do something like:

mort10 - function(lp) 1 - surv(10,lp)

and tell nomogram about mort10 using fun=.

Frank

min wrote
Hello,

I am using Dr. Harrell's rms package to make a nomogram. I was able to make a 
beautiful one. However, I want to change 5-year survival probability to 5-year 
failure probability.

I couldn’t get hazard rate from Hazard(f1) because I used cph for the model.

Here is my code:

 library(rms)
f1 - cph(Surv(retime,dfs) ~ age+her2+t_stage+n_stage+er+grade+cytcyt+Cyt_PCDK2 
 , data=data11,
surv=T, x=T, y=T, time.inc=5)

surv- Survival(f1)
haz- Hazard(f1)
Here is the Error in UseMethod(Hazard) :
  no applicable method for 'Hazard' applied to an object of class c('cph', 
'Design', 'coxph')


surv10 - function(lp) surv(10,lp)
surv5 - function(lp) surv(5,lp)
quant - Quantile(f1)

at.surv - c(0.1, 0.3, 0.5,  0.7, 0.9)

at.med1-c(2,3,4, 5,6,7,8, 10,15,20,25, 30)


par(cex=0.8)
nom- nomogram(f1, conf.int=F,
fun=list(surv5, surv10), funlabel=c('5-Year Survival Probability', '10-Year 
Survival Probability' ),  lp=F,
fun.at=c(at.surv, at.surv),label.every=1, force.label=FALSE, cex.axis=0.8, 
verbose=TRUE, cex.var=0.8)

How can I show failure probability instead of survival probability?

I would very much appreciate any assistance in this matter.
Thank you Very much.
Frank Harrell
Department of Biostatistics, Vanderbilt University


If you reply to this email, your message will be added to the discussion below:
http://r.789695.n4.nabble.com/Nomogram-with-stratified-cph-in-rms-package-how-to-get-failure-probability-tp4123249p4129173.html
To unsubscribe from Nomogram with stratified cph in rms package, how to get 
failure probability, click 
herehttp://r.789695.n4.nabble.com/template/NamlServlet.jtp?macro=unsubscribe_by_codenode=4123249code=bXlpQG1kYW5kZXJzb24ub3JnfDQxMjMyNDl8MTkzMzMxNzE4Mg==.
NAMLhttp://r.789695.n4.nabble.com/template/NamlServlet.jtp?macro=macro_viewerid=instant_html%21nabble%3Aemail.namlbase=nabble.naml.namespaces.BasicNamespace-nabble.view.web.template.NabbleNamespace-nabble.view.web.template.InstantMailNamespacebreadcrumbs=instant+emails%21nabble%3Aemail.naml-instant_emails%21nabble%3Aemail.naml-send_instant_email%21nabble%3Aemail.naml


--
View this message in context: 
http://r.789695.n4.nabble.com/Nomogram-with-stratified-cph-in-rms-package-how-to-get-failure-probability-tp4123249p4129184.html
Sent from the R help mailing list archive at Nabble.com.
[[alternative HTML version deleted]]

__
R-help@r-project.org mailing list
https://stat.ethz.ch/mailman/listinfo/r-help
PLEASE do read the posting guide http://www.R-project.org/posting-guide.html
and provide commented, minimal, self-contained, reproducible code.


Re: [R] round to specific intervals

2011-12-01 Thread Duncan Murdoch

On 01/12/2011 10:55 AM, Michael Kao wrote:

Dear R users/helpers,

I am wondering is there an existing function in which you can round
numbers to a set of values. I know you can use 5 * round(x/5) for
rounding to the nearest 5 or so, but what if the interval size is not
constant.

For example:
## Not run
test- rnorm(100)
round(test, c(1, 5, 10, 20, 50))

so that the test is rounded to the closest value in the vector.



I think you could put together such a thing using cut().

Duncan Murdoch

__
R-help@r-project.org mailing list
https://stat.ethz.ch/mailman/listinfo/r-help
PLEASE do read the posting guide http://www.R-project.org/posting-guide.html
and provide commented, minimal, self-contained, reproducible code.


Re: [R] Random Forests in R

2011-12-01 Thread Liaw, Andy
The first version of the package was created by re-writing the main program in 
the original Fortran as C, and calls other Fortran subroutines that were mostly 
untouched, so dynamic memory allocation can be done.  Later versions have most 
of the Fortran code translated/re-written in C.  Currently the only Fortran 
part is the node splitting in classification trees.

Andy

 -Original Message-
 From: r-help-boun...@r-project.org 
 [mailto:r-help-boun...@r-project.org] On Behalf Of Peter Langfelder
 Sent: Thursday, December 01, 2011 12:33 AM
 To: Axel Urbiz
 Cc: R-help@r-project.org
 Subject: Re: [R] Random Forests in R
 
 On Wed, Nov 30, 2011 at 7:48 PM, Axel Urbiz 
 axel.ur...@gmail.com wrote:
  I understand the original implementation of Random Forest 
 was done in
  Fortran code. In the source files of the R implementation 
 there is a note
  C wrapper for random forests:  get input from R and drive  
the Fortran
  routines.. I'm far from an expert on this...does that mean that the
  implementation in R is through calls to C functions only 
 (not Fortran)?
 
  So, would knowing C be enough to understand this code, or 
 Fortran is also
  necessary?
 
 I haven't seen the C and Fortran code for Random Forest but I
 understand the note to say that R code calls some C functions that
 pre-process (possibly re-format etc) the data, then call the actual
 Random Forest method that's written in Fortran, then possibly
 post-process the output and return it to R. It would imply that to
 understand the actual Random Forest code, you will have to read the
 Fortran source code.
 
 Best,
 
 Peter
 
 __
 R-help@r-project.org mailing list
 https://stat.ethz.ch/mailman/listinfo/r-help
 PLEASE do read the posting guide 
 http://www.R-project.org/posting-guide.html
 and provide commented, minimal, self-contained, reproducible code.
 
Notice:  This e-mail message, together with any attachme...{{dropped:11}}

__
R-help@r-project.org mailing list
https://stat.ethz.ch/mailman/listinfo/r-help
PLEASE do read the posting guide http://www.R-project.org/posting-guide.html
and provide commented, minimal, self-contained, reproducible code.


Re: [R] How to speed up int2bin conversion?

2011-12-01 Thread Jeff Newmiller
You might be interested in package bit.
---
Jeff NewmillerThe .   .  Go Live...
DCN:jdnew...@dcn.davis.ca.usBasics: ##.#.   ##.#.  Live Go...
  Live:   OO#.. Dead: OO#..  Playing
Research Engineer (Solar/BatteriesO.O#.   #.O#.  with
/Software/Embedded Controllers)   .OO#.   .OO#.  rocks...1k
--- 
Sent from my phone. Please excuse my brevity.

Jonas Jägermeyr jonas...@pik-potsdam.de wrote:

Dear R-help members,

I'm processing a large amount of MODIS data where quality assessment 
information is stored as an integer value for each pixel. I have to 
converted this number to an 8 digit binary flag to get access to the 
stored quality code (e.g. in2bin(165,8) = 1 0 1 0 0 1 0 1).

Unfortunately, I did not manage to find a package providing a fast 
function to do so. I need to run this on millions of pixels and thus 
wrote the following function.

int2bin - function(x,ndigits){
 base - array(NA, dim=c(length(x), ndigits))
 for(q in 1:ndigits){
   base[, ndigits-q+1] - (x %% 2)
   x - (x %/% 2)
   }
 bin- apply(base,1,paste,collapse=)
 return(bin)
}

Since it is still slow, I have to find a way to express this more 
elegantly. I'd really appreciate any help.

Thanking you, with best regards

Jonas J�germeyr

Potsdam Institute for Climate Impact Research
Research Domain II

   [[alternative HTML version deleted]]

__
R-help@r-project.org mailing list
https://stat.ethz.ch/mailman/listinfo/r-help
PLEASE do read the posting guide
http://www.R-project.org/posting-guide.html
and provide commented, minimal, self-contained, reproducible code.

__
R-help@r-project.org mailing list
https://stat.ethz.ch/mailman/listinfo/r-help
PLEASE do read the posting guide http://www.R-project.org/posting-guide.html
and provide commented, minimal, self-contained, reproducible code.


Re: [R] round to specific intervals

2011-12-01 Thread Michael Kao

Thanks for all the response,

I manage to write a small function to complete what I want based on the 
wonderful helps.


iround - function(x, interval){
## Round numbers to desired interval
##
## Args:
##   x:Numeric vector to be rounded
##   interval: The interval the values should be rounded towards.
## Retunrs:
##   a numeric vector with x rounded to the desired interval.
##
interval[ifelse(x  min(interval), 1, findInterval(x, interval))]
}

iround(0:100, c(1, 5, 10, 20, 50))

Cheers, and thanks for all the help again.


On 1/12/2011 5:20 p.m., Duncan Murdoch wrote:

On 01/12/2011 10:55 AM, Michael Kao wrote:

Dear R users/helpers,

I am wondering is there an existing function in which you can round
numbers to a set of values. I know you can use 5 * round(x/5) for
rounding to the nearest 5 or so, but what if the interval size is not
constant.

For example:
## Not run
test- rnorm(100)
round(test, c(1, 5, 10, 20, 50))

so that the test is rounded to the closest value in the vector.



I think you could put together such a thing using cut().

Duncan Murdoch


__
R-help@r-project.org mailing list
https://stat.ethz.ch/mailman/listinfo/r-help
PLEASE do read the posting guide http://www.R-project.org/posting-guide.html
and provide commented, minimal, self-contained, reproducible code.


Re: [R] how to get inflection point in binomial glm

2011-12-01 Thread David Winsemius


On Dec 1, 2011, at 8:24 AM, René Mayer wrote:


Dear All,

I have a binomial response with one continuous predictor (d) and one  
factor (g) (8 levels dummy-coded).


glm(resp~d*g, data, family=binomial)

Y=b0+b1*X1+b2*X2 ... b7*X7


Dear Dr Mayer;

I think it might be a bit more complex than that. I think you should  
get 15 betas rather than 8. Have you done it?




how can I get the inflection point per group, e.g., P(d)=.5


Wouldn't that just be at d=1/beta in each group? (Thinking, perhaps  
naively, in the case of X=X1 that


(Pr[y==1])/(1-Pr[y==1])) = 1 = exp( beta *d*(X==X1) )  # all other  
terms = 0


And taking the log of both sides, and then use middle school math to  
solve.


Oh, wait. Muffed my first try on that for sure.  Need to add back both  
the constant intercept and the baseline d coefficient for the non-b0  
levels.


(Pr[y==1])/(1-Pr[y==1])) = 1 = exp( beta_0 + beta_d_0*d +
beta_n + beta_d_n *d*(X==Xn) )

And just

(Pr[y==1])/(1-Pr[y==1])) = 1 = exp( beta_0 + beta_d_0*d ) # for the  
reference level.


This felt like an exam question in my categorical analysis course 25  
years ago. (Might have gotten partial credit for my first stab,  
depending on how forgiving the TA was that night.)




I would be grateful for any help.

Thanks in advance,
René


--

David Winsemius, MD
West Hartford, CT

__
R-help@r-project.org mailing list
https://stat.ethz.ch/mailman/listinfo/r-help
PLEASE do read the posting guide http://www.R-project.org/posting-guide.html
and provide commented, minimal, self-contained, reproducible code.


Re: [R] How to do Hotelling's t2 test?

2011-12-01 Thread S Ellison
 Hi, I want to do a 2 sample hotelling's test but i can't 
 figure out how. When i type T2.test it says there is no such 
 test and when i tried library(rrcov) it says there is no such 
 program. 

Have you installed rrcov using install.packages?

S Ellison***
This email and any attachments are confidential. Any use...{{dropped:8}}

__
R-help@r-project.org mailing list
https://stat.ethz.ch/mailman/listinfo/r-help
PLEASE do read the posting guide http://www.R-project.org/posting-guide.html
and provide commented, minimal, self-contained, reproducible code.


Re: [R] Resampling with replacement on a binary (0, 1) dataset to get Cis

2011-12-01 Thread David Winsemius


On Dec 1, 2011, at 10:49 AM, lincoln wrote:


Thanks.

So, suppose for one specific year (first year over 10) the  
percentage of
successes deriving from 100 trials with 38 successes (and 62  
failures), its

value would be 38/100=0.38.
I could calculate its confidence intervals this way:

success-38
total-100
prop.test(success,total,p=0.5,alternative=two.sided)


1-sample proportions test with continuity
correction

data:  success out of total, null probability 0.5
X-squared = 5.29, df = 1, p-value = 0.02145
alternative hypothesis: true p is not equal to 0.5
95 percent confidence interval:
0.2863947 0.4829411
sample estimates:
  p
0.38

So it would be var$1=0.38 , CI=0.286-0.483

Is it correct?



I couldn't tell if this were homework, so I just threw out a starting  
point. If you were told to do a resampling method then this would just  
be a starting point and you would be expected to go further with the  
boot function.



--
View this message in context: 
http://r.789695.n4.nabble.com/Resampling-with-replacement-on-a-binary-0-1-variable-to-get-CIs-tp4127990p4129048.html
Sent from the R help mailing list archive at Nabble.com.

__
R-help@r-project.org mailing list
https://stat.ethz.ch/mailman/listinfo/r-help
PLEASE do read the posting guide http://www.R-project.org/posting-guide.html
and provide commented, minimal, self-contained, reproducible code.


David Winsemius, MD
West Hartford, CT

__
R-help@r-project.org mailing list
https://stat.ethz.ch/mailman/listinfo/r-help
PLEASE do read the posting guide http://www.R-project.org/posting-guide.html
and provide commented, minimal, self-contained, reproducible code.


[R] legend, lheight, and alignment

2011-12-01 Thread emorway
Hello,

A bit of fairly simple code, yet I don't seem to be able to manipulate it
quite as much as I would like:

1)  It would be nice if the objects appearing in the legend were aligned,
and by aligned I mean the boxes are centered over the lines.  Do I need to
adjust the use of NA in the code below to accomplish this?  Here's how it
appears on my machine:

http://r.789695.n4.nabble.com/file/n4129402/example.png 

2)  Because I'm using a call to 'expression' in the text contained in the
legend, it would be nice to space  the lines of text a bit more.  My feeble
attempt was to increase the lheight parameter in the hopes this would affect
the legend...to no avail.  Further, lheight cannot be added to the argument
list of legend.  I've been unable to track down another parameter applicable
to legend, suggestions very much appreciated.

par(lheight=2)
plot(1,1,col=white)

legend(center, legend=c(expression(paste(Observed
,italic(bar(EC)[e]))),expression(paste(Simulated
,italic(bar(EC)[e]))),test1,test2),
   lty = c(NA,NA,1,1),
   col = c(black,black,red,blue), 
   density = c(NA,25,NA,NA),
   border=c(black,black,NA,NA),
   fill = c(grey,black,NA,NA), 
   angle = c(NA,45,NA,NA),
   cex = 1.25, bty = n, xpd = TRUE)


 sessionInfo()
R version 2.13.2 (2011-09-30)
Platform: x86_64-pc-mingw32/x64 (64-bit)

locale:
[1] LC_COLLATE=English_United States.1252 
[2] LC_CTYPE=English_United States.1252   
[3] LC_MONETARY=English_United States.1252
[4] LC_NUMERIC=C  
[5] LC_TIME=English_United States.1252

attached base packages:
[1] stats graphics  grDevices utils datasets  methods   base 

loaded via a namespace (and not attached):
[1] tools_2.13.2
 


--
View this message in context: 
http://r.789695.n4.nabble.com/legend-lheight-and-alignment-tp4129402p4129402.html
Sent from the R help mailing list archive at Nabble.com.

__
R-help@r-project.org mailing list
https://stat.ethz.ch/mailman/listinfo/r-help
PLEASE do read the posting guide http://www.R-project.org/posting-guide.html
and provide commented, minimal, self-contained, reproducible code.


Re: [R] FIML with missing data in sem package

2011-12-01 Thread yrosseel

On 12/01/2011 05:25 PM, Dustin Fife wrote:


What is your goal?  I have used and like mice pretty well, but using
mice + sem to try to address missingness seems like more work than
using FIML in OpenMx or lavaan to try to address it.  Is there a
reason you want to use the sem package or a reason you do not want to
use the others?

I tried lavaan and couldn't get it to work. I noticed it was in beta so I

figured it wasn't a good idea to use for simulations I intend to publish
(especially when I can't get them to work properly). Either way, whether I
use OpenMX or lavaan, it will require me to learn a new syntax, and I guess
I'm just lazy. I'm comfortable with sem.


The 'beta' status of lavaan implies that things (eg function arguments) 
may change from one version to another, until most planned features are 
implemented. However, the computations/results themselves are pretty 
stable. Many SEM reseachers do use lavaan today for simulation studies, 
and some have been published already. I can provide you with some 
references off-list if you are interested.


Yves Rosseel.
http://lavaan.org

__
R-help@r-project.org mailing list
https://stat.ethz.ch/mailman/listinfo/r-help
PLEASE do read the posting guide http://www.R-project.org/posting-guide.html
and provide commented, minimal, self-contained, reproducible code.


Re: [R] R, PostgresSQL and poor performance

2011-12-01 Thread Gabor Grothendieck
On Thu, Dec 1, 2011 at 10:02 AM, Berry, David I. d...@noc.ac.uk wrote:
 Hi List

 Apologies if this isn't the correct place for this query (I've tried a search 
 of the mail archives but not had much joy).

 I'm running R (2.14.0)  on a Mac (OSX v 10.5.8, 2.66GHz, 4GB memory) and am 
 having a few performance issues with reading data in from a Postres database 
 (using RPostgreSQL). My query / code are as below

 # -
 library('RPostgreSQL')

 drv - dbDriver(PostgreSQL)

 dbh - dbConnect(drv,user=…,password=…,dbname=…,host=…)

 sql - select id, date, lon, lat, date_trunc('day' , date) as jday, 
 extract('hour' from date) as hour, extract('year' from date) as year from 
 observations where pt = 6 and date = '1990-01-01' and date  '1995-01-01' 
 and lon  180 and lon  290 and lat  -30 and lat  30 and sst is not null

 dataIn - dbGetQuery(dbh,sql)

If this is a large table of which the desired rows are a small
fraction of all rows then be sure there indexes on the variables in
your where clause.

You can also try it with the RpgSQL driver although there is no reason
to think that that would be faster.

-- 
Statistics  Software Consulting
GKX Group, GKX Associates Inc.
tel: 1-877-GKX-GROUP
email: ggrothendieck at gmail.com

__
R-help@r-project.org mailing list
https://stat.ethz.ch/mailman/listinfo/r-help
PLEASE do read the posting guide http://www.R-project.org/posting-guide.html
and provide commented, minimal, self-contained, reproducible code.


[R] R package source branching

2011-12-01 Thread Mehmet Suzen
Dear List,

I was wondering if any of you worked on an R package which has many branches on 
a repository i.e. SVN.  Is it recommended to branch an R package source tree
based on a specific project? I know it depends on project but it would be great 
to 
hear opinions from R community.   

Best,

Mehmet Süzen, PhD


LEGAL NOTICE
This message is intended for the use o...{{dropped:10}}

__
R-help@r-project.org mailing list
https://stat.ethz.ch/mailman/listinfo/r-help
PLEASE do read the posting guide http://www.R-project.org/posting-guide.html
and provide commented, minimal, self-contained, reproducible code.


Re: [R] vector

2011-12-01 Thread Sarah Goslee
Hi,

On Thu, Dec 1, 2011 at 10:35 AM,  Majid  golden_boy...@yahoo.com wrote:
 Hi.
 Can you please answer to my questions about R ?
 1.how can I write command for vector ?

 for exaple in this sample :
 I have this :
 a1 - c (1:10)
 now how can I put in the vector ?

I'm afraid I don't understand your question.
a1 is a vector that you've created. What do you want to do with it?

Sarah


 bye for now,
 Thanks a lot.
 Majid.


-- 
Sarah Goslee
http://www.functionaldiversity.org

__
R-help@r-project.org mailing list
https://stat.ethz.ch/mailman/listinfo/r-help
PLEASE do read the posting guide http://www.R-project.org/posting-guide.html
and provide commented, minimal, self-contained, reproducible code.


[R] What's the baseline model when using coxph with factor variables?

2011-12-01 Thread Andreas Schlicker

Hi all,

I'm trying to fit a Cox regression model with two factor variables but 
have some problems with the interpretation of the results. Considering 
the following model, where var1 and var2 can assume value 0 and 1:


coxph(Surv(time, cens) ~ factor(var1) * factor(var2),  data=temp)

What is the baseline model? Is that considering the whole population or 
the case when both var1 and var2 = 0?


Kind regards,
andi

__
R-help@r-project.org mailing list
https://stat.ethz.ch/mailman/listinfo/r-help
PLEASE do read the posting guide http://www.R-project.org/posting-guide.html
and provide commented, minimal, self-contained, reproducible code.


Re: [R] References for book R In Action by Kabacoff (Uwe Ligges)

2011-12-01 Thread Pablo Domínguez Vaselli
The references are here: http://manning.com/kabacoff/excerpt_references.pdf

(they will be included on the next printing too, got omitted by mistake)

regards,

Pablo

[[alternative HTML version deleted]]

__
R-help@r-project.org mailing list
https://stat.ethz.ch/mailman/listinfo/r-help
PLEASE do read the posting guide http://www.R-project.org/posting-guide.html
and provide commented, minimal, self-contained, reproducible code.


[R] Counting the occurences of a charater within a string

2011-12-01 Thread Douglas Esneault
I am new to R but am experienced SAS user and I was hoping to get some help on 
counting the occurrences of a character within a string at a row level.

My dataframe, x,  is structured as below:

Col1
abc/def
ghi/jkl/mno

I found this code on the board but it counts all occurrences of / in the 
dataframe.  

chr.pos - which(unlist(strsplit(x,NULL))=='/')
chr.count - length(chr.pos)
chr.count
[1] 3

I'd like to append a column, say cnt, that has the count of / for each row.

Can anyone point me in the right direction or offer some code to do this?

Thanks in advance for the help.

Doug Esneault






Privileged/Confidential Information may be contained in this message. If you
are not the addressee indicated in this message (or responsible for delivery
of the message to such person), you may not copy or deliver this message to 
anyone. In such case, you should destroy this message and kindly notify the
sender by reply email. Please advise immediately if you or your employer
does not consent to email for messages of this kind. Opinions, conclusions
and other information in this message that do not relate to the official
business of the GroupM companies shall be understood as neither given nor
endorsed by it.   GroupM companies are a member of WPP plc. For more
information on our business ethical standards and Corporate Responsibility 
policies please refer to our website at
http://www.wpp.com/WPP/About/


__
R-help@r-project.org mailing list
https://stat.ethz.ch/mailman/listinfo/r-help
PLEASE do read the posting guide http://www.R-project.org/posting-guide.html
and provide commented, minimal, self-contained, reproducible code.


[R] Estimation of AR(1) Model with Markov Switching

2011-12-01 Thread napps22
Dear R users,

I have been trying to obtain the MLE of the following model

state 0: y_t = 2 + 0.5 * y_{t-1} + e_t
state 1: y_t = 0.5 + 0.9 * y_{t-1} + e_t

where e_t ~ iidN(0,1)

transition probability between states is 0.2

I've generated some fake data and tried to estimate the parameters using the
constrOptim() function but I can't get sensible answers using it. I've tried
using nlminb and maxLik but they haven't helped. Any tips on how I could
possibly rewrite my likelihood in a better way to improve my results would
be welcome.

Given below is my R code

# markov switching regime model
# generate data for a AR(1) markov switching model with the following pars
# state 0: y_t = 2 + 0.5 * y_{t-1} + e_t
# state 1: y_t = 0.5 + 0.9 * y_{t-1} + e_t
# where e_t ~ N(0,1) 
# transition probabilities p_s0_s1 = p_s1_s0 = 0.20

# generate realisations of the state

gamma_s0 - qnorm(0.8)
gamma_s1 - qnorm(0.2)
gamma - rep(0,100)
state - rep(0,100)

# choose initial state at t=0 to be state 0 

gamma[1] - gamma_s0
state[1] - 0

for(i in 2:100) {

if(rnorm(1)  gamma[i-1]) {
gamma[i] - gamma_s0
state[i] - 0
}
else {
gamma[i] - gamma_s1
state[i] - 1
}
}

# generate observations
# choose y_0 = 0
# recall state at t=1 was set to 0 

y1 - 2 + 0.5 * 0 + rnorm(1)
y - rep(0,100)
y[1] - y1

for(i in 2:100) {

if(state[i]==0) {
y[i] - 2 + 0.5 * y[i-1] + rnorm(1)
}
else {
y[i] - 0.5 + 0.9 * y[i-1] + rnorm(1)
}
}

# convert into time series object
 
y - ts(y, start = 1, freq = 1)

# construct negative conditional likelihood function 

neg.logl - function(theta, data) {

# construct parameters  
beta_s0 - theta[1:2]
beta_s1 - theta[3:4]
sigma2 - exp(theta[5])
gamma0 - theta[6]
gamma1 - theta[7]

# construct probabilities

#probit specification
  
p_s0_s0 - pnorm(gamma_s0)
p_s0_s1 - pnorm(gamma_s1)
p_s1_s0 - 1-pnorm(gamma_s0)
p_s1_s1 - 1-pnorm(gamma_s1)

# create data matrix

X - cbind(1,y)

# assume erogodicity of the markov chain
# use unconditional probabilities
 
p0_s0 - (1 - p_s1_s1) / (2 -p_s0_s0 -p_s1_s1)
p0_s1 - 1-p0_s0

# create variables
 
p_s0_t_1 - rep(0, nrow(X))
p_s1_t_1 - rep(0, nrow(X))
p_s0_t - rep(0, nrow(X))
p_s1_t - rep(0, nrow(X))
f_s0 - rep(0,nrow(X)-1)
f_s1 - rep(0,nrow(X)-1)
f - rep(0,nrow(X)-1)
logf - rep(0, nrow(X)-1)

p_s0_t[1] - p0_s0
p_s1_t[1] - p0_s1 

# initiate hamilton filter

for(i in 2:nrow(X)) {

# calculate prior probabilities using the TPT
# TPT for this example gives us 
# p_si_t_1 = p_si_t_1_si * p_si_t + p_si_t_1_sj * p_si_t
# where p_si_t_1 is the prob state_t = i given information @ time t-1
# p_si_t_1_sj is the prob state_t = i given state_t_1 = j, and all info @
time t-1
# p_si_t is the prob state_t = i given information @ time t

# in this simple example p_si_t_1_sj = p_si_sj

p_s0_t_1[i] - (p_s0_s0 * p_s0_t[i-1]) + (p_s0_s1 * p_s1_t[i-1])
p_s1_t_1[i] - (p_s1_s0 * p_s0_t[i-1]) + (p_s1_s1 * p_s1_t[i-1])

# calculate density function for observation i
# f_si is density conditional on state = i
# f is the density 
  
f_s0[i] - dnorm(y[i]-X[i-1,]%*%beta_s0, sd = sqrt(sigma2)) 
f_s1[i] - dnorm(y[i]-X[i-1,]%*%beta_s1, sd = sqrt(sigma2))
f[i] - (f_s0[i] * p_s0_t_1[i]) + (f_s1[i] * p_s1_t_1[i])

# calculate filtered/posterior probabilities using bayes rule
# p_si_t is the prob that state = i given information @ time t

p_s0_t[i] - (f_s0[i] * p_s0_t_1[i]) / f[i]
p_s1_t[i] - (f_s1[i] * p_s1_t_1[i]) / f[i]

logf[i] - log(f[i])

}

logl -sum(logf)
return(-logl)

}


# restrict intercept in state model 0 to be greater than intercept in state
model 1
# thus matrix of restrictions R is [1 0 -1 0 0 0 0]

R - matrix(c(1,0,-1,0,0,0,0), nrow = 1)

# pick start values for the 7 unknown parameters

start_val - matrix(runif(7), nrow = 7)

# ensures starting values are in the feasible set 

start_val[1,] - start_val[3,] + 0.1

# estimate pars 

results -constrOptim(start_val,neg.logl,grad = NULL, ui = R, ci = 0)

Regards,

N

--
View this message in context: 
http://r.789695.n4.nabble.com/Estimation-of-AR-1-Model-with-Markov-Switching-tp4129417p4129417.html
Sent from the R help mailing list archive at Nabble.com.

__
R-help@r-project.org mailing list
https://stat.ethz.ch/mailman/listinfo/r-help
PLEASE do read the posting guide http://www.R-project.org/posting-guide.html
and provide commented, minimal, self-contained, reproducible code.


Re: [R] bigmemory on Solaris

2011-12-01 Thread RogerP
Thanks again for your help.

I've been able to add several packages, bigmemory seems to be the only one
to fail and it 
fails on isinf.  

Is there a way I can download the code and change it to include a ininf
function or definition?

I'm using the GNU compiler; should I have been using the SUN Studio
compiler?

Roger

--
View this message in context: 
http://r.789695.n4.nabble.com/bigmemory-on-Solaris-tp4128179p4129290.html
Sent from the R help mailing list archive at Nabble.com.

__
R-help@r-project.org mailing list
https://stat.ethz.ch/mailman/listinfo/r-help
PLEASE do read the posting guide http://www.R-project.org/posting-guide.html
and provide commented, minimal, self-contained, reproducible code.


Re: [R] What's the baseline model when using coxph with factor variables?

2011-12-01 Thread David Winsemius


On Dec 1, 2011, at 12:00 PM, Andreas Schlicker wrote:


Hi all,

I'm trying to fit a Cox regression model with two factor variables  
but have some problems with the interpretation of the results.  
Considering the following model, where var1 and var2 can assume  
value 0 and 1:


coxph(Surv(time, cens) ~ factor(var1) * factor(var2),  data=temp)

What is the baseline model? Is that considering the whole population  
or the case when both var1 and var2 = 0?


This has been discussed several times in the past on rhelp. My  
suggestion would be to search your favorite rhelp archive using   
baseline hazard Therneau, since Terry Therneau is the author of  
survival. (The answer is closer to the first than to the second.)




Kind regards,
andi

__
R-help@r-project.org mailing list
https://stat.ethz.ch/mailman/listinfo/r-help
PLEASE do read the posting guide http://www.R-project.org/posting-guide.html
and provide commented, minimal, self-contained, reproducible code.


David Winsemius, MD
West Hartford, CT

__
R-help@r-project.org mailing list
https://stat.ethz.ch/mailman/listinfo/r-help
PLEASE do read the posting guide http://www.R-project.org/posting-guide.html
and provide commented, minimal, self-contained, reproducible code.


[R] logistic regression - glm.fit: fitted probabilities numerically 0 or 1 occurred

2011-12-01 Thread Ben quant
Sorry if this is a duplicate: This is a re-post because the pdf's mentioned
below did not go through.

Hello,

I'm new'ish to R, and very new to glm. I've read a lot about my issue:
Warning message:
glm.fit: fitted probabilities numerically 0 or 1 occurred

...including:

http://tolstoy.newcastle.edu.au/R/help/05/07/7759.html
http://r.789695.n4.nabble.com/glm-fit-quot-fitted-probabilities-numerically-0-or-1-occurred-quot-td849242.html
(note that I never found: MASS4 pp.197-8  However, Ted's post was quite
helpful.)

This is a common question, sorry. Because it is a common issue I am posting
everything I know about the issue and how I think I am not falling into the
same trap at the others (but I must be due to some reason I am not yet
aware of).

From the two links above I gather that my warning glm.fit: fitted
probabilities numerically 0 or 1 occurred arises from a perfect fit
situation (i.e. the issue where all the high value x's (predictor
variables) are Y=1 (response=1) or the other way around). I don't feel my
data has this issue. Please point out how it does!

The list post instructions state that I can attach pdf's, so I attached
plots of my data right before I do the call to glm.

The attachments are plots of my data stored in variable l_yx (as can be
seen in the axis names):
My response (vertical axis) by row index (horizontal axis):
 plot(l_yx[,1],type='h')
My predictor variable (vertical axis) by row index index (horizontal axis):
 plot(l_yx[,2],type='h')

 So here is more info on my data frame/data (in case you can't see my pdf
attachments):
 unique(l_yx[,1])
[1] 0 1
 mean(l_yx[,2])
[1] 0.01123699
 max(l_yx[,2])
[1] 14.66518
 min(l_yx[,2])
[1] 0
 attributes(l_yx)
$dim
[1] 690303  2

$dimnames
$dimnames[[1]]
NULL

$dimnames[[2]]
[1] y x


With the above data I do:
 l_logit = glm(y~x, data=as.data.frame(l_yx),
family=binomial(link=logit))
Warning message:
glm.fit: fitted probabilities numerically 0 or 1 occurred

Why am I getting this warning when I have data points of varying values for
y=1 and y=0?  In other words, I don't think I have the linear separation
issue discussed in one of the links I provided.

PS - Then I do this and I get a odds ratio a crazy size:
 l_sm = summary(l_logit) # coef pval is $coefficients[8], log odds
$coefficients[2]
 l_exp_coef = exp(l_logit$coefficients)[2] # exponentiate the
coeffcients
 l_exp_coef
   x
3161.781

So for one unit increase in the predictor variable I get 3160.781%
(3161.781 - 1 = 3160.781) increase in odds? That can't be correct either.
How do I correct for this issue? (I tried multiplying the predictor
variables by a constant and the odds ratio goes down, but the warning above
still persists and shouldn't the odds ratio be predictor variable size
independent?)

Thank you for your help!

Ben

[[alternative HTML version deleted]]

__
R-help@r-project.org mailing list
https://stat.ethz.ch/mailman/listinfo/r-help
PLEASE do read the posting guide http://www.R-project.org/posting-guide.html
and provide commented, minimal, self-contained, reproducible code.


Re: [R] What's the baseline model when using coxph with factor variables?

2011-12-01 Thread William Dunlap
Terry will correct me if I'm wrong, but I don't think the
answer to this question is specific to the coxph function.
For all the [well-written] formula-based modelling functions
(essentially, those that call model.frame and model.matrix to interpret
the formula) the option contrasts controls how factor
variables are parameterized in the model matrix.  contr.treatment
makes the baseline the first factor level, contr.SAS makes
the baseline the last, contr.sum makes the baseline the mean,
etc.  E.g.,

 df - data.frame(time=sin(1:20)+2,
   cens=rep(c(0,0,1), len=20),
   var1=factor(rep(0:1, each=10)),
   var2=factor(rep(0:1, 10))) 
 options(contrasts=c(contr.treatment, contr.treatment))
 coxph(Surv(time, cens) ~ var1 + var2, data=df)
Call:
coxph(formula = Surv(time, cens) ~ var1 + var2, data = df)


coef exp(coef) se(coef)  zp
var11 0.1640  1.180.822 0.1995 0.84
var21 0.0806  1.080.830 0.0971 0.92

Likelihood ratio test=0.05  on 2 df, p=0.974  n= 20, number of events= 6 
 options(contrasts=c(contr.SAS, contr.SAS))
 coxph(Surv(time, cens) ~ var1 + var2, data=df)
Call:
coxph(formula = Surv(time, cens) ~ var1 + var2, data = df)


 coef exp(coef) se(coef)   zp
var10 -0.1640 0.8490.822 -0.1995 0.84
var20 -0.0806 0.9230.830 -0.0971 0.92

Likelihood ratio test=0.05  on 2 df, p=0.974  n= 20, number of events= 6 
 options(contrasts=c(contr.sum, contr.sum))
 coxph(Surv(time, cens) ~ var1 + var2, data=df)
Call:
coxph(formula = Surv(time, cens) ~ var1 + var2, data = df)


 coef exp(coef) se(coef)   zp
var11 -0.0820 0.9210.411 -0.1995 0.84
var21 -0.0403 0.9600.415 -0.0971 0.92

Likelihood ratio test=0.05  on 2 df, p=0.974  n= 20, number of events= 6

(lm() has a contrasts argument that can override getOption(contrasts)
and set different contrasts for each variable but coxph() does not have
that argument.)

Bill Dunlap
Spotfire, TIBCO Software
wdunlap tibco.com 

 -Original Message-
 From: r-help-boun...@r-project.org [mailto:r-help-boun...@r-project.org] On 
 Behalf Of David Winsemius
 Sent: Thursday, December 01, 2011 9:36 AM
 To: a.schlic...@nki.nl
 Cc: r-help@r-project.org
 Subject: Re: [R] What's the baseline model when using coxph with factor 
 variables?
 
 
 On Dec 1, 2011, at 12:00 PM, Andreas Schlicker wrote:
 
  Hi all,
 
  I'm trying to fit a Cox regression model with two factor variables
  but have some problems with the interpretation of the results.
  Considering the following model, where var1 and var2 can assume
  value 0 and 1:
 
  coxph(Surv(time, cens) ~ factor(var1) * factor(var2),  data=temp)
 
  What is the baseline model? Is that considering the whole population
  or the case when both var1 and var2 = 0?
 
 This has been discussed several times in the past on rhelp. My
 suggestion would be to search your favorite rhelp archive using
 baseline hazard Therneau, since Terry Therneau is the author of
 survival. (The answer is closer to the first than to the second.)
 
 
  Kind regards,
  andi
 
  __
  R-help@r-project.org mailing list
  https://stat.ethz.ch/mailman/listinfo/r-help
  PLEASE do read the posting guide http://www.R-project.org/posting-guide.html
  and provide commented, minimal, self-contained, reproducible code.
 
 David Winsemius, MD
 West Hartford, CT
 
 __
 R-help@r-project.org mailing list
 https://stat.ethz.ch/mailman/listinfo/r-help
 PLEASE do read the posting guide http://www.R-project.org/posting-guide.html
 and provide commented, minimal, self-contained, reproducible code.

__
R-help@r-project.org mailing list
https://stat.ethz.ch/mailman/listinfo/r-help
PLEASE do read the posting guide http://www.R-project.org/posting-guide.html
and provide commented, minimal, self-contained, reproducible code.


Re: [R] Counting the occurences of a charater within a string

2011-12-01 Thread Bert Gunter
## It's not a data frame -- it's just a vector.

 x
[1] abc/def ghi/jkl/mno
 gsub([^/],,x)
[1] /  //
 nchar(gsub([^/],,x))
[1] 1 2


?gsub
?nchar

-- Bert

On Thu, Dec 1, 2011 at 8:32 AM, Douglas Esneault
douglas.esnea...@mecglobal.com wrote:
 I am new to R but am experienced SAS user and I was hoping to get some help 
 on counting the occurrences of a character within a string at a row level.

 My dataframe, x,  is structured as below:

 Col1
 abc/def
 ghi/jkl/mno

 I found this code on the board but it counts all occurrences of / in the 
 dataframe.

 chr.pos - which(unlist(strsplit(x,NULL))=='/')
 chr.count - length(chr.pos)
 chr.count
 [1] 3

 I'd like to append a column, say cnt, that has the count of / for each row.

 Can anyone point me in the right direction or offer some code to do this?

 Thanks in advance for the help.

 Doug Esneault






 Privileged/Confidential Information may be contained in this message. If you
 are not the addressee indicated in this message (or responsible for delivery
 of the message to such person), you may not copy or deliver this message to
 anyone. In such case, you should destroy this message and kindly notify the
 sender by reply email. Please advise immediately if you or your employer
 does not consent to email for messages of this kind. Opinions, conclusions
 and other information in this message that do not relate to the official
 business of the GroupM companies shall be understood as neither given nor
 endorsed by it.   GroupM companies are a member of WPP plc. For more
 information on our business ethical standards and Corporate Responsibility
 policies please refer to our website at
 http://www.wpp.com/WPP/About/


 __
 R-help@r-project.org mailing list
 https://stat.ethz.ch/mailman/listinfo/r-help
 PLEASE do read the posting guide http://www.R-project.org/posting-guide.html
 and provide commented, minimal, self-contained, reproducible code.



-- 

Bert Gunter
Genentech Nonclinical Biostatistics

Internal Contact Info:
Phone: 467-7374
Website:
http://pharmadevelopment.roche.com/index/pdb/pdb-functional-groups/pdb-biostatistics/pdb-ncb-home.htm

__
R-help@r-project.org mailing list
https://stat.ethz.ch/mailman/listinfo/r-help
PLEASE do read the posting guide http://www.R-project.org/posting-guide.html
and provide commented, minimal, self-contained, reproducible code.


Re: [R] Writing a function, want a string argument to define the name of the excel sheet to be called

2011-12-01 Thread R. Michael Weylandt
Just a heads up -- I don't think your code will work with an actual
.xls(x) file, only .txt, .csv, etc (aka, plain text files). I may be
wrong about that, but if you actually need to work with Excel files
directly you will need an additional package.

Michael

On Thu, Dec 1, 2011 at 9:10 AM, AOLeary aodha...@gmail.com wrote:
 Thank you very much, that does exactly what I want it to! :)

 Aodhán

 --
 View this message in context: 
 http://r.789695.n4.nabble.com/Writing-a-function-want-a-string-argument-to-define-the-name-of-the-excel-sheet-to-be-called-tp4128384p4128495.html
 Sent from the R help mailing list archive at Nabble.com.

 __
 R-help@r-project.org mailing list
 https://stat.ethz.ch/mailman/listinfo/r-help
 PLEASE do read the posting guide http://www.R-project.org/posting-guide.html
 and provide commented, minimal, self-contained, reproducible code.

__
R-help@r-project.org mailing list
https://stat.ethz.ch/mailman/listinfo/r-help
PLEASE do read the posting guide http://www.R-project.org/posting-guide.html
and provide commented, minimal, self-contained, reproducible code.


[R] transform data.frame holding answers -- data.frame holding logicals

2011-12-01 Thread saschaview

Hello

Hello

I have a data frame, x, holding 5 persons answering the question which 
cars they have used:


# the data frame
x - as.data.frame(
  matrix(
c('BMW', '', '',
  'Mercedes', 'VW', '',
  'Skoda', 'VW', 'BMW',
  '', '', '',
  'VW', 'Skoda', ''
),
ncol=3,
dimnames=list(
  NULL,
  paste( 'v', 1:3, sep='' )
)
  )
)

How do I transform this data frame to a data frame stating whether they 
have used the presented car:


   BMW  Mercedes  VW  Skoda
1  TF F   F
2  FT T   F
3  TF T   T
4  FF F   F
5  FF T   T

My idea was to first find all levels by:

v - unique(unlist(lapply(x, levels)))

But then I am stuck.

Thanks for help, *S*

--
Sascha Vieweg, saschav...@gmail.com

__
R-help@r-project.org mailing list
https://stat.ethz.ch/mailman/listinfo/r-help
PLEASE do read the posting guide http://www.R-project.org/posting-guide.html
and provide commented, minimal, self-contained, reproducible code.


Re: [R] Replace columns in a data.frame randomly splitted

2011-12-01 Thread R. Michael Weylandt
I repeat myself:

Any more automated solution will depend on whether your data has
rownames or not. [...] create a plain text representation of R data
using the dput() command.

Another way that might make more sense is to cbind() the data you need
later on before the split and then it will be carried through.

Michael

On Thu, Dec 1, 2011 at 2:54 AM, agent dunham crossp...@hotmail.com wrote:


 The thing is that I've already been working with df1, and I was looking for
 a function that could replace values knowing rows and columns. Does it
 exist?

 Thank you, u...@host.com

 --
 View this message in context: 
 http://r.789695.n4.nabble.com/Replace-columns-in-a-data-frame-randomly-splitted-tp4122926p4127405.html
 Sent from the R help mailing list archive at Nabble.com.

 __
 R-help@r-project.org mailing list
 https://stat.ethz.ch/mailman/listinfo/r-help
 PLEASE do read the posting guide http://www.R-project.org/posting-guide.html
 and provide commented, minimal, self-contained, reproducible code.

__
R-help@r-project.org mailing list
https://stat.ethz.ch/mailman/listinfo/r-help
PLEASE do read the posting guide http://www.R-project.org/posting-guide.html
and provide commented, minimal, self-contained, reproducible code.


Re: [R] logistic regression - glm.fit: fitted probabilities numerically 0 or 1 occurred

2011-12-01 Thread peter dalgaard

On Dec 1, 2011, at 18:54 , Ben quant wrote:

 Sorry if this is a duplicate: This is a re-post because the pdf's mentioned
 below did not go through.

Still not there. Sometimes it's because your mailer doesn't label them with the 
appropriate mime-type (e.g. as application/octet-stream, which is arbitrary 
binary). Anyways, see below

[snip]
 
 With the above data I do:
l_logit = glm(y~x, data=as.data.frame(l_yx),
 family=binomial(link=logit))
 Warning message:
 glm.fit: fitted probabilities numerically 0 or 1 occurred
 
 Why am I getting this warning when I have data points of varying values for
 y=1 and y=0?  In other words, I don't think I have the linear separation
 issue discussed in one of the links I provided.

I bet that you do... You can get the warning without that effect (one of my own 
examples is  the probability of menarche in a data set that includes infants 
and old age pensioners), but not with a huge odds ratio as well. Take a look at 

d - as.data.frame(l_yx) 
with(d, y[order(x)])

if it comes out as all zeros followed by all ones or vice versa, then you have 
the problem.


 
 PS - Then I do this and I get a odds ratio a crazy size:
l_sm = summary(l_logit) # coef pval is $coefficients[8], log odds
 $coefficients[2]
l_exp_coef = exp(l_logit$coefficients)[2] # exponentiate the
 coeffcients
l_exp_coef
   x
 3161.781
 
 So for one unit increase in the predictor variable I get 3160.781%
 (3161.781 - 1 = 3160.781) increase in odds? That can't be correct either.
 How do I correct for this issue? (I tried multiplying the predictor
 variables by a constant and the odds ratio goes down, but the warning above
 still persists and shouldn't the odds ratio be predictor variable size
 independent?)


-- 
Peter Dalgaard, Professor,
Center for Statistics, Copenhagen Business School
Solbjerg Plads 3, 2000 Frederiksberg, Denmark
Phone: (+45)38153501
Email: pd@cbs.dk  Priv: pda...@gmail.com

__
R-help@r-project.org mailing list
https://stat.ethz.ch/mailman/listinfo/r-help
PLEASE do read the posting guide http://www.R-project.org/posting-guide.html
and provide commented, minimal, self-contained, reproducible code.


[R] Change the limits of a plot a posteriori

2011-12-01 Thread jcano
Hi all

How can I change the limits (xlim or ylim) in a plot that has been already
created?

For example, consider this naive example 
curve(dbeta(x,2,4))
curve(dbeta(x,8,13),add=T,col=2)

When adding the second curve, it goes off the original limits computed by R
for the first graph, which are roughly, c(0,2.1)

I know two obvious solutions for this, which are:
1) passing a sufficiently large parameter e.g. ylim=c(0,4) to the first
graphic
curve(dbeta(x,2,4),ylim=c(0,4))
curve(dbeta(x,8,13),add=T,col=2)

or 

2) switch the order in which I plot the curves
curve(dbeta(x,8,13),col=2)
curve(dbeta(x,2,4),add=T)

but I guess if there is any way of adjusting the limits of the graphic a
posteriori, once you have a plot with the undesired limits, forcing R to
redraw it with the new limits, but without having to execute again the
curve commands

Hope I made myself clear

Best regards and thank you very much in advance


--
View this message in context: 
http://r.789695.n4.nabble.com/Change-the-limits-of-a-plot-a-posteriori-tp4129750p4129750.html
Sent from the R help mailing list archive at Nabble.com.

__
R-help@r-project.org mailing list
https://stat.ethz.ch/mailman/listinfo/r-help
PLEASE do read the posting guide http://www.R-project.org/posting-guide.html
and provide commented, minimal, self-contained, reproducible code.


[R] Assign name to object for each iteration in a loop.

2011-12-01 Thread lglew
Hi R-users,

I'm trying to produce decompositions of a multiple time-series, grouped by a
factor (called area). I'm modifying the code in the STLperArea function of
package ndvits, as this function only plots produces stl plots, it does not
return the underlying data. 

I want to extract the trend component of each decomposition
(x$time.series[,trend]), assign a name based on the factor area. 

My input data look like this:
Area is a factor, with three (but could be many more) levels.
area
1
2
3

Ystart=2000

TS is a timeseries:

  X249   X265  X281  X297  X2000113 
1 0.2080  0.2165  0.2149 0.2314  0.2028  
2 0.1578  0.1671  0.1577 0.1593  0.1672   
3 0.1897  0.1948  0.2290 0.2292  0.2067   

Here's the function:

STLpA-function(TS, area, Ystart, period=23, nSG=5,5, DSG=0)
{
require (RTisean)
for(i in 1:unique(area)){
vi.metric=TS[area==i]
filt.vi-sav_gol(vi.metric,n=nSG,D=DSG)
vi.sg-ts(filt.vi[,1], start=Ystart,frequency=period)
stld.tmp-stl(vi.sg, s.window=periodic, robust=TRUE, na.action=na.approx)
stld.trend-stld.temp$time.series[,trend]
}
assign(paste(stld, i , sep= .), stld.trend)
vi.trend-ls(pattern= ^stld..$)
return(vi.trend)
}

When I call this function with signal=STLpA(TS,area,Ystart=2000,period=23,
nSG= 5,5, DSG=0))

I get this error:

Error in cat(list(...), file, sep, fill, labels, append) : 
  argument 1 (type 'list') cannot be handled by 'cat'
In addition: Warning message:
In 1:unique(area) :
  numerical expression has 3 elements: only the first used

I'm guessing this is because I'm assigning names to each temporary stl.trend
file incorrectly. Can anyone 
improve on my rather poor efforts here?

Many thanks,

Louise




--
View this message in context: 
http://r.789695.n4.nabble.com/Assign-name-to-object-for-each-iteration-in-a-loop-tp4129752p4129752.html
Sent from the R help mailing list archive at Nabble.com.

__
R-help@r-project.org mailing list
https://stat.ethz.ch/mailman/listinfo/r-help
PLEASE do read the posting guide http://www.R-project.org/posting-guide.html
and provide commented, minimal, self-contained, reproducible code.


[R] nested random effects with lmer

2011-12-01 Thread Charles Li
Hi,

 

I have a multilevel situation where subjects are nested within clinics, and
each subject has multiple measurements.

For simplicity, suppose there 4 clinics, 3 subjects per clinic, and each
subject has 3 repeated measures.

Outcome is continuous. I am trying to implement this model with lmer
function in lme4 library.

 

lmeobj1-lmer(outcome~med+time+medtime+(time|clinid)+(subjid|clinid)) 

 

My data set is as follows: med has two levels, time has three levels,
medtime is the interaction between med and time, subjid is the id numbers
for subjects,

clinid is the id numbers for clinics. I pasted the data below.

 

The above code is giving the same results with fixed effects regression. In
addition, whatever I put as random effects

is not making a difference at all. What am I doing wrong?

 

cbind(outcome,med,time,medtime,subjid,clinid)

 

 outcome   med  time medtime subjid  clinid

[1,]0.8511317  0   0   010 4

[2,]3.9524773  0   1   010 4

[3,]2.1637332  0   2   010 4

[4,]1.6713403  1   0   0 1 1

[5,]1.7379582  1   1   1 1 1

[6,]5.0951544  1   2   2 1 1

[7,]0.9523377  1   0   0 2 1

[8,]2.4599613  1   1   1 2 1

[9,]5.1759744  1   2   2 2 1

[10,]   -0.9495338  1   0   0 4 2

[11,]2.9300264  1   1   1 4 2

[12,]3.7025651  1   2   2 4 2

[13,]1.1628554  1   0   0 6 2

[14,]4.1536130  1   1   1 6 2

[15,]5.0869188  1   2   2 6 2

[16,]0.3114241  0   0   0 7 3

[17,]0.6007636  0   1   0 7 3

[18,]2.2783941  0   2   0 7 3

[19,]   -0.7095164  0   0   0 8 3

[20,]   -0.1541448  0   1   0 8 3

[21,]1.6037837  0   2   0 8 3

[22,]1.9994632  0   0   0 9 3

[23,]0.8047261  0   1   0 9 3

[24,]0.1828438  0   2   0 9 3

[25,]2.7219649  0   0   012 4

[26,]2.3046305  0   1   012 4

[27,]1.1838549  0   2   012 4

[28,]0.6292358  1   0   0 3 1

[29,]2.9953366  1   1   1 3 1

[30,]3.9291266  1   2   2 3 1

[31,]0.8726671  0   0   011 4

[32,]   -0.4657651  0   1   011 4

[33,]3.0364098  0   2   011 4

[34,]0.3046405  1   0   0 5 2

[35,]0.8789185  1   1   1 5 2

[36,]4.1729769  1   2   2 5 2

 

Thanks,


[[alternative HTML version deleted]]

__
R-help@r-project.org mailing list
https://stat.ethz.ch/mailman/listinfo/r-help
PLEASE do read the posting guide http://www.R-project.org/posting-guide.html
and provide commented, minimal, self-contained, reproducible code.


[R] question about plsr() results

2011-12-01 Thread Vytautas Rakeviius
Hi,
With some help I learned how to use plsr(Y~x, 2, data=my) function in R (and 
build my from vector Y and matrix x). 

But still I have question about results interpretation. In the end I want to 
construct prediction function in form:
Y=a1x1+a2x2
But I do not understand how to do it. Documentation do not describe this.
Thanks for help. ;)

[[alternative HTML version deleted]]

__
R-help@r-project.org mailing list
https://stat.ethz.ch/mailman/listinfo/r-help
PLEASE do read the posting guide http://www.R-project.org/posting-guide.html
and provide commented, minimal, self-contained, reproducible code.


Re: [R] Change the limits of a plot a posteriori

2011-12-01 Thread Jean V Adams
jcano wrote on 12/01/2011 12:12:03 PM:

 Hi all
 
 How can I change the limits (xlim or ylim) in a plot that has been 
already
 created?
 
 For example, consider this naive example 
 curve(dbeta(x,2,4))
 curve(dbeta(x,8,13),add=T,col=2)
 
 When adding the second curve, it goes off the original limits computed 
by R
 for the first graph, which are roughly, c(0,2.1)
 
 I know two obvious solutions for this, which are:
 1) passing a sufficiently large parameter e.g. ylim=c(0,4) to the first
 graphic
 curve(dbeta(x,2,4),ylim=c(0,4))
 curve(dbeta(x,8,13),add=T,col=2)
 
 or 
 
 2) switch the order in which I plot the curves
 curve(dbeta(x,8,13),col=2)
 curve(dbeta(x,2,4),add=T)
 
 but I guess if there is any way of adjusting the limits of the graphic 
a
 posteriori, once you have a plot with the undesired limits, forcing R 
to
 redraw it with the new limits, but without having to execute again the
 curve commands
 
 Hope I made myself clear
 
 Best regards and thank you very much in advance


There is no way to adjust the limits of the plot a posteriori.
You could set it up so that the limits of the plot are determined directly 
from the data you are plotting.

x - seq(0, 1, 0.01)
y.range - range(dbeta(x, 2, 4), dbeta(x, 8, 13))
curve(dbeta(x, 2, 4), ylim=y.range) 
curve(dbeta(x, 8, 13), ylim=y.range, add=T, col=2) 

Jean
[[alternative HTML version deleted]]

__
R-help@r-project.org mailing list
https://stat.ethz.ch/mailman/listinfo/r-help
PLEASE do read the posting guide http://www.R-project.org/posting-guide.html
and provide commented, minimal, self-contained, reproducible code.


Re: [R] how to get inflection point in binomial glm

2011-12-01 Thread René Mayer

Thanks David and Rubén!

@David: indeed 15 betas I forgot the interaction terms, thanks for correcting!

@Rubén:  the re-parameterize would be done within nls()? how to do  
this practically with including the factor predictor?


and yes, we can solve within each group for Y=0 getting

0=b0+b1*X |-b0
-b0=b1*X |/b1
-b0/b1=X

but I was hoping there might a more general solution for the case of  
multiple logistic regression.



HTH

René

Zitat von David Winsemius dwinsem...@comcast.net:



On Dec 1, 2011, at 8:24 AM, René Mayer wrote:


Dear All,

I have a binomial response with one continuous predictor (d) and  
one factor (g) (8 levels dummy-coded).


glm(resp~d*g, data, family=binomial)

Y=b0+b1*X1+b2*X2 ... b7*X7


Dear Dr Mayer;

I think it might be a bit more complex than that. I think you should  
get 15 betas rather than 8. Have you done it?




how can I get the inflection point per group, e.g., P(d)=.5


Wouldn't that just be at d=1/beta in each group? (Thinking, perhaps  
naively, in the case of X=X1 that


(Pr[y==1])/(1-Pr[y==1])) = 1 = exp( beta *d*(X==X1) )  # all other terms = 0

And taking the log of both sides, and then use middle school math to solve.

Oh, wait. Muffed my first try on that for sure.  Need to add back  
both the constant intercept and the baseline d coefficient for the  
non-b0 levels.


(Pr[y==1])/(1-Pr[y==1])) = 1 = exp( beta_0 + beta_d_0*d +
beta_n + beta_d_n *d*(X==Xn) )

And just

(Pr[y==1])/(1-Pr[y==1])) = 1 = exp( beta_0 + beta_d_0*d ) # for the  
reference level.


This felt like an exam question in my categorical analysis course 25  
years ago. (Might have gotten partial credit for my first stab,  
depending on how forgiving the TA was that night.)




I would be grateful for any help.

Thanks in advance,
René


--

David Winsemius, MD
West Hartford, CT




__
R-help@r-project.org mailing list
https://stat.ethz.ch/mailman/listinfo/r-help
PLEASE do read the posting guide http://www.R-project.org/posting-guide.html
and provide commented, minimal, self-contained, reproducible code.


Re: [R] Assign name to object for each iteration in a loop.

2011-12-01 Thread R. Michael Weylandt
I think part of your problem is the loop:

you probably mean

for(i in unique(area))

Michael

On Thu, Dec 1, 2011 at 1:13 PM, lglew l.g...@soton.ac.uk wrote:
 Hi R-users,

 I'm trying to produce decompositions of a multiple time-series, grouped by a
 factor (called area). I'm modifying the code in the STLperArea function of
 package ndvits, as this function only plots produces stl plots, it does not
 return the underlying data.

 I want to extract the trend component of each decomposition
 (x$time.series[,trend]), assign a name based on the factor area.

 My input data look like this:
 Area is a factor, with three (but could be many more) levels.
 area
 1
 2
 3

 Ystart=2000

 TS is a timeseries:

  X249   X265  X281  X297  X2000113
 1     0.2080      0.2165      0.2149     0.2314      0.2028
 2     0.1578      0.1671      0.1577     0.1593      0.1672
 3     0.1897      0.1948      0.2290     0.2292      0.2067

 Here's the function:

 STLpA-function(TS, area, Ystart, period=23, nSG=5,5, DSG=0)
 {
 require (RTisean)
 for(i in 1:unique(area)){
 vi.metric=TS[area==i]
 filt.vi-sav_gol(vi.metric,n=nSG,D=DSG)
 vi.sg-ts(filt.vi[,1], start=Ystart,frequency=period)
 stld.tmp-stl(vi.sg, s.window=periodic, robust=TRUE, na.action=na.approx)
 stld.trend-stld.temp$time.series[,trend]
 }
 assign(paste(stld, i , sep= .), stld.trend)
 vi.trend-ls(pattern= ^stld..$)
 return(vi.trend)
 }

 When I call this function with signal=STLpA(TS,area,Ystart=2000,period=23,
 nSG= 5,5, DSG=0))

 I get this error:

 Error in cat(list(...), file, sep, fill, labels, append) :
  argument 1 (type 'list') cannot be handled by 'cat'
 In addition: Warning message:
 In 1:unique(area) :
  numerical expression has 3 elements: only the first used

 I'm guessing this is because I'm assigning names to each temporary stl.trend
 file incorrectly. Can anyone
 improve on my rather poor efforts here?

 Many thanks,

 Louise




 --
 View this message in context: 
 http://r.789695.n4.nabble.com/Assign-name-to-object-for-each-iteration-in-a-loop-tp4129752p4129752.html
 Sent from the R help mailing list archive at Nabble.com.

 __
 R-help@r-project.org mailing list
 https://stat.ethz.ch/mailman/listinfo/r-help
 PLEASE do read the posting guide http://www.R-project.org/posting-guide.html
 and provide commented, minimal, self-contained, reproducible code.

__
R-help@r-project.org mailing list
https://stat.ethz.ch/mailman/listinfo/r-help
PLEASE do read the posting guide http://www.R-project.org/posting-guide.html
and provide commented, minimal, self-contained, reproducible code.


Re: [R] Change the limits of a plot a posteriori

2011-12-01 Thread Duncan Murdoch

On 01/12/2011 1:12 PM, jcano wrote:

Hi all

How can I change the limits (xlim or ylim) in a plot that has been already
created?


You can't, if you're using classic R graphics.  They use an ink on 
paper model of graphics. If you want to change what you've drawn, you 
get a new piece of paper.


Your two solutions below are the usual methods to work around this 
limitation.   The first is the easiest method in general, though it's 
not particularly easy if you're using curve().   Generally if you're 
planning to plot y1 vs x1 and y2 vs x2, you can set your ylim to 
range(c(y1, y2)) and your xlim to range(c(x1, x2)) and things are fine.  
If you want to follow this strategy with curve(), you need to call it 
twice for each curve:  once to find the range of points it would plot 
(they are in the return value of the function), and a second time to 
redo the plot with the calculated xlim and ylim.


Duncan Murdoch


For example, consider this naive example
curve(dbeta(x,2,4))
curve(dbeta(x,8,13),add=T,col=2)

When adding the second curve, it goes off the original limits computed by R
for the first graph, which are roughly, c(0,2.1)

I know two obvious solutions for this, which are:
1) passing a sufficiently large parameter e.g. ylim=c(0,4) to the first
graphic
curve(dbeta(x,2,4),ylim=c(0,4))
curve(dbeta(x,8,13),add=T,col=2)

or

2) switch the order in which I plot the curves
curve(dbeta(x,8,13),col=2)
curve(dbeta(x,2,4),add=T)

but I guess if there is any way of adjusting the limits of the graphic a
posteriori, once you have a plot with the undesired limits, forcing R to
redraw it with the new limits, but without having to execute again the
curve commands

Hope I made myself clear

Best regards and thank you very much in advance


--
View this message in context: 
http://r.789695.n4.nabble.com/Change-the-limits-of-a-plot-a-posteriori-tp4129750p4129750.html
Sent from the R help mailing list archive at Nabble.com.

__
R-help@r-project.org mailing list
https://stat.ethz.ch/mailman/listinfo/r-help
PLEASE do read the posting guide http://www.R-project.org/posting-guide.html
and provide commented, minimal, self-contained, reproducible code.


__
R-help@r-project.org mailing list
https://stat.ethz.ch/mailman/listinfo/r-help
PLEASE do read the posting guide http://www.R-project.org/posting-guide.html
and provide commented, minimal, self-contained, reproducible code.


Re: [R] transform data.frame holding answers -- data.frame holding logicals

2011-12-01 Thread Jean V Adams
saschaview wrote on 12/01/2011 12:30:18 PM:

 Hello
 
 I have a data frame, x, holding 5 persons answering the question which 
 cars they have used:
 
 # the data frame
 x - as.data.frame(
matrix(
  c('BMW', '', '',
'Mercedes', 'VW', '',
'Skoda', 'VW', 'BMW',
'', '', '',
'VW', 'Skoda', ''
  ),
  ncol=3,
  dimnames=list(
NULL,
paste( 'v', 1:3, sep='' )
  )
)
 )
 
 How do I transform this data frame to a data frame stating whether they 
 have used the presented car:
 
 BMW  Mercedes  VW  Skoda
 1  TF F   F
 2  FT T   F
 3  TF T   T
 4  FF F   F
 5  FF T   T
 
 My idea was to first find all levels by:
 
 v - unique(unlist(lapply(x, levels)))
 
 But then I am stuck.
 
 Thanks for help, *S*
 
 -- 
 Sascha Vieweg, saschav...@gmail.com


Try this:

uniq.cars - sort(unique(unlist(x)))
y - t(apply(x, 1, function(xrow) match(uniq.cars, unique(xrow
dimnames(y)[[2]] - uniq.cars
y2 - !is.na(y[, -1])
y2

Jean
[[alternative HTML version deleted]]

__
R-help@r-project.org mailing list
https://stat.ethz.ch/mailman/listinfo/r-help
PLEASE do read the posting guide http://www.R-project.org/posting-guide.html
and provide commented, minimal, self-contained, reproducible code.


Re: [R] vector

2011-12-01 Thread Sarah Goslee
Hi,

On Thu, Dec 1, 2011 at 2:43 PM,  Majid  golden_boy...@yahoo.com wrote:
 Dear Sarah.
 Thanks so much,Really I am new in this software,I am wrking to learn the
 software.

First, you should always send your replies to the list. That way
information can help others,
and more people are available to provide advice.

Second, is this homework?

Third, see below.

 I have this sample and I was checking it :
 #Generating data===

 # in here we produce patterned data
 #Do this and see what will happen:

 1:10

 # now put this seri in a vector

 a1 - c (1:10)

 #==



 #Do this and see what will happen:
 seq(1, 5, 0.5)

 # now put this seri in a vector

 a2 - c (seq(1, 5, 0.5))

 Can you please learn me what is the matter  about these orders ?

Nothing is wrong with these commands. Both make vectors. The c() is
unnecessary in both cases, but still works.

If this is not homework, you still need to explain what you *expect*
to have happen, otherwise nobody can help you.

Sarah

-- 
Sarah Goslee
http://www.functionaldiversity.org

__
R-help@r-project.org mailing list
https://stat.ethz.ch/mailman/listinfo/r-help
PLEASE do read the posting guide http://www.R-project.org/posting-guide.html
and provide commented, minimal, self-contained, reproducible code.


Re: [R] What's the baseline model when using coxph with factor variables?

2011-12-01 Thread David Winsemius


On Dec 1, 2011, at 1:00 PM, William Dunlap wrote:


Terry will correct me if I'm wrong, but I don't think the
answer to this question is specific to the coxph function.


It depends on our interpretation of the questioner's intent. My answer  
was predicated on the assumption that the phrase baseline model  
meant baseline survival function, ... S_0(t) in survival analysis  
notation.




For all the [well-written] formula-based modelling functions
(essentially, those that call model.frame and model.matrix to  
interpret

the formula) the option contrasts controls how factor
variables are parameterized in the model matrix.  contr.treatment
makes the baseline the first factor level, contr.SAS makes
the baseline the last, contr.sum makes the baseline the mean,
etc.  E.g.,


df - data.frame(time=sin(1:20)+2,

  cens=rep(c(0,0,1), len=20),
  var1=factor(rep(0:1, each=10)),
  var2=factor(rep(0:1, 10)))

options(contrasts=c(contr.treatment, contr.treatment))
coxph(Surv(time, cens) ~ var1 + var2, data=df)

Call:
coxph(formula = Surv(time, cens) ~ var1 + var2, data = df)


   coef exp(coef) se(coef)  zp
var11 0.1640  1.180.822 0.1995 0.84
var21 0.0806  1.080.830 0.0971 0.92

Likelihood ratio test=0.05  on 2 df, p=0.974  n= 20, number of  
events= 6

options(contrasts=c(contr.SAS, contr.SAS))
coxph(Surv(time, cens) ~ var1 + var2, data=df)

Call:
coxph(formula = Surv(time, cens) ~ var1 + var2, data = df)


coef exp(coef) se(coef)   zp
var10 -0.1640 0.8490.822 -0.1995 0.84
var20 -0.0806 0.9230.830 -0.0971 0.92

Likelihood ratio test=0.05  on 2 df, p=0.974  n= 20, number of  
events= 6

options(contrasts=c(contr.sum, contr.sum))
coxph(Surv(time, cens) ~ var1 + var2, data=df)

Call:
coxph(formula = Surv(time, cens) ~ var1 + var2, data = df)


coef exp(coef) se(coef)   zp
var11 -0.0820 0.9210.411 -0.1995 0.84
var21 -0.0403 0.9600.415 -0.0971 0.92

Likelihood ratio test=0.05  on 2 df, p=0.974  n= 20, number of  
events= 6


(lm() has a contrasts argument that can override  
getOption(contrasts)
and set different contrasts for each variable but coxph() does not  
have

that argument.)

Bill Dunlap
Spotfire, TIBCO Software
wdunlap tibco.com


-Original Message-
From: r-help-boun...@r-project.org [mailto:r-help-boun...@r-project.org 
] On Behalf Of David Winsemius

Sent: Thursday, December 01, 2011 9:36 AM
To: a.schlic...@nki.nl
Cc: r-help@r-project.org
Subject: Re: [R] What's the baseline model when using coxph with  
factor variables?



On Dec 1, 2011, at 12:00 PM, Andreas Schlicker wrote:


Hi all,

I'm trying to fit a Cox regression model with two factor variables
but have some problems with the interpretation of the results.
Considering the following model, where var1 and var2 can assume
value 0 and 1:

coxph(Surv(time, cens) ~ factor(var1) * factor(var2),  data=temp)

What is the baseline model? Is that considering the whole population
or the case when both var1 and var2 = 0?


This has been discussed several times in the past on rhelp. My
suggestion would be to search your favorite rhelp archive using
baseline hazard Therneau, since Terry Therneau is the author of
survival. (The answer is closer to the first than to the second.)



Kind regards,
andi

__
R-help@r-project.org mailing list
https://stat.ethz.ch/mailman/listinfo/r-help
PLEASE do read the posting guide http://www.R-project.org/posting-guide.html
and provide commented, minimal, self-contained, reproducible code.


David Winsemius, MD
West Hartford, CT

__
R-help@r-project.org mailing list
https://stat.ethz.ch/mailman/listinfo/r-help
PLEASE do read the posting guide http://www.R-project.org/posting-guide.html
and provide commented, minimal, self-contained, reproducible code.


David Winsemius, MD
West Hartford, CT

__
R-help@r-project.org mailing list
https://stat.ethz.ch/mailman/listinfo/r-help
PLEASE do read the posting guide http://www.R-project.org/posting-guide.html
and provide commented, minimal, self-contained, reproducible code.


Re: [R] vector

2011-12-01 Thread Sarah Goslee
On Thu, Dec 1, 2011 at 3:11 PM,  Majid  golden_boy...@yahoo.com wrote:
 Hi,
 yes, It is homework,

Then ask your TA/instructor for help.


 These are 2 command:
 first for generating data:
 (1:10)
 that output is 1 2 310
 ok ?
 second is :
 a1-c( 1:10)
 what is the output ?I didnot see any thing.

Exactly as it should be. There's no problem here, except that you need to read
help(-)


 Thanks,
 Majid.


-- 
Sarah Goslee
http://www.functionaldiversity.org

__
R-help@r-project.org mailing list
https://stat.ethz.ch/mailman/listinfo/r-help
PLEASE do read the posting guide http://www.R-project.org/posting-guide.html
and provide commented, minimal, self-contained, reproducible code.


Re: [R] logistic regression - glm.fit: fitted probabilities numerically 0 or 1 occurred

2011-12-01 Thread Ben quant
Thank you for the feedback, but my data looks fine to me. Please tell me if
I'm not understanding.

I followed your instructions and here is a sample of the first 500 values :
(info on 'd' is below that)

  d - as.data.frame(l_yx)
 x = with(d, y[order(x)])
 x[1:500] # I have 1's and 0's dispersed throughout
  [1] 0 0 0 0 0 0 0 0 0 0 0 0 0 0 0 0 0 0 0 0 0 0 0 0 0 0 0 0 0 0 0 0 0 0 0
0 0 0 0 0 0 0 0 0 0 0 0 0 0 0 0 0 0 0 0 0 0 0 0 0 0 0 0 0 0 0 0 0 0 0 0 0 0
0 0 0 0 0 0 0 0 0 0 0 0 0 1 1 1 1 0 0 0 0 0 0 0 0 0 0
[101] 0 0 0 0 0 0 0 0 0 0 0 0 0 0 0 0 0 0 0 0 0 0 0 0 0 0 0 0 0 0 0 0 0 0 0
0 0 0 0 0 0 0 0 0 0 0 0 0 0 0 0 0 0 0 0 0 0 0 0 0 0 0 0 0 0 0 0 0 0 0 0 0 0
0 0 1 0 0 0 0 0 0 0 0 0 0 0 0 0 0 0 0 0 0 0 0 0 0 0 0
[201] 0 0 0 0 0 0 0 0 0 0 0 0 0 0 0 0 0 0 0 0 1 0 0 0 0 0 0 0 0 0 0 0 0 1 0
0 0 0 0 0 0 0 1 0 0 0 0 1 0 0 0 0 0 1 0 0 0 0 0 0 0 0 0 0 0 0 0 1 0 0 0 0 0
1 0 0 0 0 0 0 0 1 0 0 0 0 0 0 1 0 1 0 0 1 0 0 0 1 0 0
[301] 1 0 0 0 0 0 0 0 0 0 0 0 0 0 0 0 1 0 0 0 0 0 0 0 0 0 0 0 0 0 0 0 0 0 0
0 0 1 0 0 0 0 0 0 0 0 0 0 0 0 0 0 0 0 0 0 1 1 0 0 0 0 0 0 0 0 1 0 1 0 0 0 0
0 0 0 0 0 0 0 0 0 0 0 0 0 0 0 0 0 0 0 0 0 0 0 0 0 0 0
[401] 0 0 0 0 0 0 0 0 0 0 0 0 0 0 0 0 0 0 0 0 0 0 0 0 0 1 1 0 0 0 0 0 0 0 0
1 0 0 0 0 0 0 0 0 0 0 0 0 0 0 0 0 0 1 0 0 0 0 0 0 0 0 0 0 0 0 0 0 0 0 0 0 0
0 0 0 0 0 0 0 0 0 0 0 0 0 0 0 0 0 0 0 0 0 0 1 0 0 0 0

# I get the warning still
 l_df = as.data.frame(l_yx)
 l_logit = glm(y~x, data=l_df, family=binomial(link=logit))

Warning message:
glm.fit: fitted probabilities numerically 0 or 1 occurred

# some info on 'd' above:

 d[1:500,1]
  [1] 0 0 0 0 0 0 0 0 0 0 0 0 0 0 0 0 0 0 0 0 0 0 0 0 0 0 0 0 0 0 0 0 0 0 0
0 0 0 0 0 0 0 0 0 0 0 0 0 0 0 0 0 0 0 0 0 0 0 0 0 0 0 0 0 0 0 0 0 0 0 0 0 0
0 0 0 0 0 0 0 0 0 0 0 0 0 0 0 0 0 0 0 0 0 0 0 0 0 0 0
[101] 0 0 0 0 0 0 0 0 0 0 0 0 0 0 0 0 0 0 0 0 0 0 0 0 0 0 0 0 0 0 0 0 0 0 0
0 0 0 0 0 0 0 0 0 0 0 0 0 0 0 0 0 0 0 0 0 0 0 0 0 0 0 0 0 0 0 0 0 0 0 0 0 0
0 0 0 0 0 0 0 0 0 0 0 0 0 0 0 0 0 0 0 0 0 0 0 0 0 0 0
[201] 0 0 0 0 0 0 0 0 0 0 0 0 0 0 0 0 0 0 0 0 0 0 0 0 0 0 0 0 0 0 0 0 0 0 0
0 0 0 0 0 0 0 0 0 0 0 0 0 0 0 0 0 0 0 0 0 0 0 0 0 0 0 0 0 0 0 0 0 0 0 0 0 0
0 0 0 0 0 0 0 0 0 0 0 0 0 0 0 0 0 0 0 0 0 0 0 0 0 0 0
[301] 0 0 0 0 0 0 0 0 0 0 0 0 0 0 0 0 0 0 0 0 0 0 0 0 0 0 0 0 0 0 0 0 0 0 0
0 0 0 0 0 0 0 0 0 0 0 0 0 0 0 0 0 0 0 0 0 0 0 0 0 0 0 0 0 0 0 0 0 0 0 0 0 0
0 0 0 0 0 0 0 0 0 0 0 0 0 0 0 0 0 0 0 0 0 0 0 0 0 0 0
[401] 0 0 0 0 0 0 0 0 0 0 0 0 0 0 0 0 0 0 0 0 0 0 0 0 0 0 0 0 0 0 0 0 0 0 0
0 0 0 0 0 0 0 0 0 0 0 0 0 0 0 0 0 0 0 0 0 0 0 0 0 0 0 0 0 0 0 0 0 0 0 0 0 0
0 0 0 0 0 0 0 0 0 0 0 0 0 0 0 0 0 0 0 0 0 0 0 0 0 0 0
 d[1:500,2]
  [1] 3.023160e-03 7.932130e-02 0.00e+00 1.779657e-02 1.608374e-01
0.00e+00 5.577064e-02 7.753926e-03 4.018553e-03 4.760918e-02
2.080511e-02 1.642404e-01 3.703720e-03 8.901981e-02 1.260415e-03
 [16] 2.202523e-02 3.750940e-02 4.441975e-04 9.351171e-03 8.374567e-03
0.00e+00 8.440448e-02 5.081017e-01 2.538640e-05 1.806017e-02
2.954641e-04 1.434859e-03 6.964976e-04 0.00e+00 1.202162e-02
 [31] 3.420300e-03 4.276100e-02 1.457324e-02 4.140121e-03 1.349180e-04
1.525292e-03 4.817502e-02 9.515717e-03 2.232953e-02 1.227908e-01
3.293581e-02 1.454352e-02 1.176011e-03 6.274138e-02 2.879205e-02
 [46] 6.900926e-03 1.414648e-04 3.446349e-02 8.807174e-03 3.549332e-02
2.828509e-03 2.935003e-02 7.162872e-03 5.650050e-03 1.221191e-02
0.00e+00 2.126334e-02 2.052020e-02 7.542409e-02 2.586269e-04
 [61] 5.258664e-02 1.213126e-02 1.493275e-02 8.152657e-03 1.774966e-02
2.433198e-02 1.371257e-02 6.732339e-02 9.747677e-03 8.180594e-03
1.882473e-01 1.682596e-03 1.410170e-02 8.429676e-03 3.520589e-01
 [76] 1.865160e-02 2.389941e-02 3.301558e-02 4.896605e-03 6.826244e-03
9.151337e-02 1.722270e-02 1.764039e-01 3.554063e-02 7.338532e-03
9.577793e-03 3.632366e-03 1.422400e-02 3.020497e-02 1.797096e-02
 [91] 2.577899e-03 1.338365e-01 8.202223e-03 1.088335e-02 2.137259e-02
6.829797e-03 3.104860e-04 8.927160e-03 5.626881e-02 1.197682e-02
6.140553e-02 2.729364e-02 1.377913e-02 3.018277e-03 1.188304e-02
[106] 2.029268e-03 2.108815e-02 1.765503e-03 2.449253e-02 3.677046e-03
1.013463e-02 4.286642e-02 1.238931e-02 3.072090e-03 1.509613e-02
3.609885e-02 5.537268e-03 3.151614e-02 3.703148e-03 1.409401e-01
[121] 1.473365e-02 9.160461e-03 1.035099e-01 3.005865e-02 1.332199e-02
6.936535e-03 2.433539e-02 4.935373e-03 4.822740e-03 1.597643e-03
3.805619e-03 1.092683e-02 1.760635e-01 5.565614e-03 7.739213e-04
[136] 4.529198e-03 5.110359e-03 7.391258e-02 5.018207e-03 2.071417e-02
1.825787e-02 9.141405e-03 1.078386e-01 2.171470e-04 7.164583e-03
2.522077e-02 1.994428e-03 6.044653e-03 1.778078e-04 5.797706e-03
[151] 1.526778e-02 1.595897e-02 1.995627e-01 1.946735e-03 6.711582e-02
2.722350e-02 3.127499e-02 1.074904e-01 2.477414e-03 5.015375e-02
3.417810e-02 2.046643e-02 1.644832e-02 5.220166e-03 1.217752e-02
[166] 1.187352e-02 1.634016e-02 2.349568e-03 3.451811e-02 2.593540e-03
6.868595e-03 1.311236e-02 6.457757e-03 2.942391e-04 1.628352e-02
8.288831e-03 3.170856e-04 

[R] calculate mean of multiple rows in a data frame

2011-12-01 Thread Jabez Wilson
Dear all, I have a data frame (DF) in the following format:










NAME
ID
a
b
c
d

1
Control_1
probe~B01R01C01
381
213
345
653

2
Control_2
probe~B01R01C02
574
629
563
783

3
Control_1
probe~B01R09C01
673
511
521
967

4
Control_3
probe~B01R09C02
53
809
999
50

5
MM0289~RFU:11810.15
probe~B29R13C06
681
34
115
587

6
MM0289~RFU:9238.41
probe~B29R13C05
784
443
20
784

7
MM16597~RFU:36765.38
probe~B44R15C20
719
251
790
445

8
MM16597~RFU:41258.94
probe~B44R15C19
677
363
268
686.
I would like to consolidate the data frame by parsing through the rows, and 
where the NAME is identical, consolidate into one row and return the mean.
I can do this for the first lines (Control_1 etc) by using aggregate()
aggregate(DF[,-c(1:2)], by=list(DF$NAME), mean)
but since aggregate looks for unique lines it won't consolidate e.g. lines 5/6 
and 7/8.
Is there a way of telling aggregate to grep just the first part of the name 
(i.e. up to ~) and consolidate those?
I could pre-grep the file before importing into R, but I'd like to do it within 
R if possible.
Thanks for any suggestions
 
[[alternative HTML version deleted]]

__
R-help@r-project.org mailing list
https://stat.ethz.ch/mailman/listinfo/r-help
PLEASE do read the posting guide http://www.R-project.org/posting-guide.html
and provide commented, minimal, self-contained, reproducible code.


Re: [R] combining arima and ar function

2011-12-01 Thread R. Michael Weylandt
I think your attachment got scrubbed so I can't verify this, but i
think the difficulty is that the inner apply returns a whole set of
orders, instead of just one and that throws arima off: would this
work?

getOrder - function(x) ar(na.omit(x), method = mle)$order

all.equal(apply(TSX, 2, getOrder), apply(TSX,2, function(x)
ar(na.omit(x),method=mle)$order) # Verify it works.

apply(TSX,2,function(x)arima(x,order=c(getOrder(x),0,0))$residuals

Michael

On Thu, Dec 1, 2011 at 4:57 AM, Samir Benzerfa benze...@gmx.ch wrote:
 Hi everyone



 I've got a problem regarding the arima() and the ar() function for
 autoregressive series. I would simply like to combine them.



 To better understand my question, I first show you how I'm using these two
 functions individually (see file in the attachement).



 1)      apply(TSX,2, function(x) ar(na.omit(x),method=mle)$order
 # this function finds the optimal (autoregressive) order for each series
 (briefly speaking: for each series I get a specific number)

 2)      apply(TSX,2,function(x)arima(x,order=c(1,0,0))$residuals
 # this function puts an autoregressive model of order 1 on each series





 What I'd like to do, is to combine them, such that the resulting orders
 (numbers) from the first function are used as orders in the second function.
 So in the specific example attached the results from the first function are
 6,5,1 and these numbers should be used as the order in function two. I tried
 to simply put the two functions together as follows:



 apply(TSX,2,function(x) arima(x,order=c((apply(TSX,2,
 function(x)ar(na.omit(x),method=mle)$order))),0,0))



 However, I get the error message  'order' must be a non-negative numeric
 vector of length 3.



 Any hints? I hope you can help me with this issue.



 Many thanks and best regards, S.B.








 __
 R-help@r-project.org mailing list
 https://stat.ethz.ch/mailman/listinfo/r-help
 PLEASE do read the posting guide http://www.R-project.org/posting-guide.html
 and provide commented, minimal, self-contained, reproducible code.


__
R-help@r-project.org mailing list
https://stat.ethz.ch/mailman/listinfo/r-help
PLEASE do read the posting guide http://www.R-project.org/posting-guide.html
and provide commented, minimal, self-contained, reproducible code.


[R] Fw: calculate mean of multiple rows in a data frame

2011-12-01 Thread Jabez Wilson



NAME
ID
a
b
c
d

1
Control_1
probe~B01R01C01
381
213
345
653

2
Control_2
probe~B01R01C02
574
629
563
783

3
Control_1
probe~B01R09C01
673
511
521
967

4
Control_3
probe~B01R09C02
53
809
999
50

5
MM0289~RFU:11810.15
probe~B29R13C06
681
34
115
587

6
MM0289~RFU:9238.41
probe~B29R13C05
784
443
20
784

7
MM16597~RFU:36765.38
probe~B44R15C20
719
251
790
445

8
MM16597~RFU:41258.94
probe~B44R15C19
677
363
268
686



NAME
ID
a
b
c
d

1
Control_1
probe~B01R01C01
381
213
345
653

2
Control_2
probe~B01R01C02
574
629
563
783

3
Control_1
probe~B01R09C01
673
511
521
967

4
Control_3
probe~B01R09C02
53
809
999
50

5
MM0289~RFU:11810.15
probe~B29R13C06
681
34
115
587

6
MM0289~RFU:9238.41
probe~B29R13C05
784
443
20
784

7
MM16597~RFU:36765.38
probe~B44R15C20
719
251
790
445

8
MM16597~RFU:41258.94
probe~B44R15C19
677
363
268
686
Sorry, that should look like this:




NAME
ID
a
b
c
d

1
Control_1
probe~B01R01C01
381
213
345
653

2
Control_2
probe~B01R01C02
574
629
563
783

3
Control_1
probe~B01R09C01
673
511
521
967

4
Control_3
probe~B01R09C02
53
809
999
50

5
MM0289~RFU:11810.15
probe~B29R13C06
681
34
115
587

6
MM0289~RFU:9238.41
probe~B29R13C05
784
443
20
784

7
MM16597~RFU:36765.38
probe~B44R15C20
719
251
790
445

8
MM16597~RFU:41258.94
probe~B44R15C19
677
363
268
686 NAME ID a b c d 
1 Control_1 probe~B01R01C01 3 22 926 774 
2 Control_2 probe~B01R01C02 712 13 32 179 
3 Control_1 probe~B01R09C01 937 824 898 668 
4 Control_3 probe~B01R09C02 464 836 508 53 
5 MM0289~RFU:11810.15 probe~B29R13C06 99 544 607 984 
6 MM0289~RFU:9238.41 probe~B29R13C05 605 603 862 575 
7 MM16597~RFU:36765.38 probe~B44R15C20 700 923 219 582 
8 MM16597~RFU:41258.94 probe~B44R15C19 132 777 497 995

--- On Thu, 1/12/11, Jabez Wilson jabez...@yahoo.co.uk wrote:


From: Jabez Wilson jabez...@yahoo.co.uk
Subject: calculate mean of multiple rows in a data frame
To: R-Help r-h...@stat.math.ethz.ch
Date: Thursday, 1 December, 2011, 20:45







Dear all, I have a data frame (DF) in the following format:










NAME
ID
a
b
c
d

1
Control_1
probe~B01R01C01
381
213
345
653

2
Control_2
probe~B01R01C02
574
629
563
783

3
Control_1
probe~B01R09C01
673
511
521
967

4
Control_3
probe~B01R09C02
53
809
999
50

5
MM0289~RFU:11810.15
probe~B29R13C06
681
34
115
587

6
MM0289~RFU:9238.41
probe~B29R13C05
784
443
20
784

7
MM16597~RFU:36765.38
probe~B44R15C20
719
251
790
445

8
MM16597~RFU:41258.94
probe~B44R15C19
677
363
268
686.
I would like to consolidate the data frame by parsing through the rows, and 
where the NAME is identical, consolidate into one row and return the mean.
I can do this for the first lines (Control_1 etc) by using aggregate()
aggregate(DF[,-c(1:2)], by=list(DF$NAME), mean)
but since aggregate looks for unique lines it won't consolidate e.g. lines 5/6 
and 7/8.
Is there a way of telling aggregate to grep just the first part of the name 
(i.e. up to ~) and consolidate those?
I could pre-grep the file before importing into R, but I'd like to do it within 
R if possible.
Thanks for any suggestions
 
[[alternative HTML version deleted]]

__
R-help@r-project.org mailing list
https://stat.ethz.ch/mailman/listinfo/r-help
PLEASE do read the posting guide http://www.R-project.org/posting-guide.html
and provide commented, minimal, self-contained, reproducible code.


Re: [R] logistic regression - glm.fit: fitted probabilities numerically 0 or 1 occurred

2011-12-01 Thread peter dalgaard

On Dec 1, 2011, at 21:32 , Ben quant wrote:

 Thank you for the feedback, but my data looks fine to me. Please tell me if 
 I'm not understanding.

Hum, then maybe it really is a case of a transition region being short relative 
to the range of your data. Notice that the warning is just that: a warning. I 
do notice that the distribution of your x values is rather extreme -- you 
stated a range of 0--14 and a mean of 0.01. And after all, an odds ratio of 
3000 per unit is only a tad over a doubling per 0.1 units.

Have a look at  range(x[y==0]) and range(x[y==1]). 


 
 I followed your instructions and here is a sample of the first 500 values : 
 (info on 'd' is below that)
 
   d - as.data.frame(l_yx) 
  x = with(d, y[order(x)])
  x[1:500] # I have 1's and 0's dispersed throughout 
   [1] 0 0 0 0 0 0 0 0 0 0 0 0 0 0 0 0 0 0 0 0 0 0 0 0 0 0 0 0 0 0 0 0 0 0 0 0 
 0 0 0 0 0 0 0 0 0 0 0 0 0 0 0 0 0 0 0 0 0 0 0 0 0 0 0 0 0 0 0 0 0 0 0 0 0 0 0 
 0 0 0 0 0 0 0 0 0 0 0 1 1 1 1 0 0 0 0 0 0 0 0 0 0
 [101] 0 0 0 0 0 0 0 0 0 0 0 0 0 0 0 0 0 0 0 0 0 0 0 0 0 0 0 0 0 0 0 0 0 0 0 0 
 0 0 0 0 0 0 0 0 0 0 0 0 0 0 0 0 0 0 0 0 0 0 0 0 0 0 0 0 0 0 0 0 0 0 0 0 0 0 0 
 1 0 0 0 0 0 0 0 0 0 0 0 0 0 0 0 0 0 0 0 0 0 0 0 0
 [201] 0 0 0 0 0 0 0 0 0 0 0 0 0 0 0 0 0 0 0 0 1 0 0 0 0 0 0 0 0 0 0 0 0 1 0 0 
 0 0 0 0 0 0 1 0 0 0 0 1 0 0 0 0 0 1 0 0 0 0 0 0 0 0 0 0 0 0 0 1 0 0 0 0 0 1 0 
 0 0 0 0 0 0 1 0 0 0 0 0 0 1 0 1 0 0 1 0 0 0 1 0 0
 [301] 1 0 0 0 0 0 0 0 0 0 0 0 0 0 0 0 1 0 0 0 0 0 0 0 0 0 0 0 0 0 0 0 0 0 0 0 
 0 1 0 0 0 0 0 0 0 0 0 0 0 0 0 0 0 0 0 0 1 1 0 0 0 0 0 0 0 0 1 0 1 0 0 0 0 0 0 
 0 0 0 0 0 0 0 0 0 0 0 0 0 0 0 0 0 0 0 0 0 0 0 0 0
 [401] 0 0 0 0 0 0 0 0 0 0 0 0 0 0 0 0 0 0 0 0 0 0 0 0 0 1 1 0 0 0 0 0 0 0 0 1 
 0 0 0 0 0 0 0 0 0 0 0 0 0 0 0 0 0 1 0 0 0 0 0 0 0 0 0 0 0 0 0 0 0 0 0 0 0 0 0 
 0 0 0 0 0 0 0 0 0 0 0 0 0 0 0 0 0 0 0 0 1 0 0 0 0
 
 # I get the warning still
  l_df = as.data.frame(l_yx)
  l_logit = glm(y~x, data=l_df, family=binomial(link=logit))
 
 Warning message:
 glm.fit: fitted probabilities numerically 0 or 1 occurred 
 
 # some info on 'd' above:
 
  d[1:500,1]
   [1] 0 0 0 0 0 0 0 0 0 0 0 0 0 0 0 0 0 0 0 0 0 0 0 0 0 0 0 0 0 0 0 0 0 0 0 0 
 0 0 0 0 0 0 0 0 0 0 0 0 0 0 0 0 0 0 0 0 0 0 0 0 0 0 0 0 0 0 0 0 0 0 0 0 0 0 0 
 0 0 0 0 0 0 0 0 0 0 0 0 0 0 0 0 0 0 0 0 0 0 0 0 0
 [101] 0 0 0 0 0 0 0 0 0 0 0 0 0 0 0 0 0 0 0 0 0 0 0 0 0 0 0 0 0 0 0 0 0 0 0 0 
 0 0 0 0 0 0 0 0 0 0 0 0 0 0 0 0 0 0 0 0 0 0 0 0 0 0 0 0 0 0 0 0 0 0 0 0 0 0 0 
 0 0 0 0 0 0 0 0 0 0 0 0 0 0 0 0 0 0 0 0 0 0 0 0 0
 [201] 0 0 0 0 0 0 0 0 0 0 0 0 0 0 0 0 0 0 0 0 0 0 0 0 0 0 0 0 0 0 0 0 0 0 0 0 
 0 0 0 0 0 0 0 0 0 0 0 0 0 0 0 0 0 0 0 0 0 0 0 0 0 0 0 0 0 0 0 0 0 0 0 0 0 0 0 
 0 0 0 0 0 0 0 0 0 0 0 0 0 0 0 0 0 0 0 0 0 0 0 0 0
 [301] 0 0 0 0 0 0 0 0 0 0 0 0 0 0 0 0 0 0 0 0 0 0 0 0 0 0 0 0 0 0 0 0 0 0 0 0 
 0 0 0 0 0 0 0 0 0 0 0 0 0 0 0 0 0 0 0 0 0 0 0 0 0 0 0 0 0 0 0 0 0 0 0 0 0 0 0 
 0 0 0 0 0 0 0 0 0 0 0 0 0 0 0 0 0 0 0 0 0 0 0 0 0
 [401] 0 0 0 0 0 0 0 0 0 0 0 0 0 0 0 0 0 0 0 0 0 0 0 0 0 0 0 0 0 0 0 0 0 0 0 0 
 0 0 0 0 0 0 0 0 0 0 0 0 0 0 0 0 0 0 0 0 0 0 0 0 0 0 0 0 0 0 0 0 0 0 0 0 0 0 0 
 0 0 0 0 0 0 0 0 0 0 0 0 0 0 0 0 0 0 0 0 0 0 0 0 0
  d[1:500,2]
   [1] 3.023160e-03 7.932130e-02 0.00e+00 1.779657e-02 1.608374e-01 
 0.00e+00 5.577064e-02 7.753926e-03 4.018553e-03 4.760918e-02 2.080511e-02 
 1.642404e-01 3.703720e-03  8.901981e-02 1.260415e-03
  [16] 2.202523e-02 3.750940e-02 4.441975e-04 9.351171e-03 8.374567e-03 
 0.00e+00 8.440448e-02 5.081017e-01 2.538640e-05 1.806017e-02 2.954641e-04 
 1.434859e-03 6.964976e-04 0.00e+00 1.202162e-02
  [31] 3.420300e-03 4.276100e-02 1.457324e-02 4.140121e-03 1.349180e-04 
 1.525292e-03 4.817502e-02 9.515717e-03 2.232953e-02 1.227908e-01 3.293581e-02 
 1.454352e-02 1.176011e-03 6.274138e-02 2.879205e-02
  [46] 6.900926e-03 1.414648e-04 3.446349e-02 8.807174e-03 3.549332e-02 
 2.828509e-03 2.935003e-02 7.162872e-03 5.650050e-03 1.221191e-02 0.00e+00 
 2.126334e-02 2.052020e-02 7.542409e-02 2.586269e-04
  [61] 5.258664e-02 1.213126e-02 1.493275e-02 8.152657e-03 1.774966e-02 
 2.433198e-02 1.371257e-02 6.732339e-02 9.747677e-03 8.180594e-03 1.882473e-01 
 1.682596e-03 1.410170e-02 8.429676e-03 3.520589e-01
  [76] 1.865160e-02 2.389941e-02 3.301558e-02 4.896605e-03 6.826244e-03 
 9.151337e-02 1.722270e-02 1.764039e-01 3.554063e-02 7.338532e-03 9.577793e-03 
 3.632366e-03 1.422400e-02 3.020497e-02 1.797096e-02
  [91] 2.577899e-03 1.338365e-01 8.202223e-03 1.088335e-02 2.137259e-02 
 6.829797e-03 3.104860e-04 8.927160e-03 5.626881e-02 1.197682e-02 6.140553e-02 
 2.729364e-02 1.377913e-02 3.018277e-03 1.188304e-02
 [106] 2.029268e-03 2.108815e-02 1.765503e-03 2.449253e-02 3.677046e-03 
 1.013463e-02 4.286642e-02 1.238931e-02 3.072090e-03 1.509613e-02 3.609885e-02 
 5.537268e-03 3.151614e-02 3.703148e-03 1.409401e-01
 [121] 1.473365e-02 9.160461e-03 1.035099e-01 3.005865e-02 1.332199e-02 
 6.936535e-03 2.433539e-02 4.935373e-03 4.822740e-03 1.597643e-03 3.805619e-03 
 1.092683e-02 1.760635e-01 

[R] MCMCglmm error with multinomial distribution

2011-12-01 Thread Håvard Wahl Kongsgård
With binomial/binary responses (0|1) running MCMCglmm with
family=multinomial terminates with

Error in if (nJ  1) { : missing value where TRUE/FALSE needed

with family=categorical there are no errors

I have not looked in the code, do I need format the responses
TRUE/FALSE , or is this just a bug?

-- 
Håvard Wahl Kongsgård

__
R-help@r-project.org mailing list
https://stat.ethz.ch/mailman/listinfo/r-help
PLEASE do read the posting guide http://www.R-project.org/posting-guide.html
and provide commented, minimal, self-contained, reproducible code.


  1   2   >